Download as pdf or txt
Download as pdf or txt
You are on page 1of 83

Total Marks : 200

TEST - 3 (Textbook)
( INSTA Prelims Test Series 2021 )

1. How are standardized Phase-III vaccine trials different from Phase-II and Phase-I trials?
1. Phase-III trials are double-blind trials that use Placebo, whereas Phase-I and II do not.
2. Phase-III trials include diseased people, whereas Phase-I and II include only completely
healthy people.
3. Phase-III trials have a larger and more diverse target group than the other two phases.

Select the correct answer using the codes below.


A. 2 and 3 only
B. 1 only
C. 3 only
D. 1, 2 and 3

Correct Answer : C

Answer Justification :

Justification: Phase 0 of a clinical trial is done with a very small number of people, usually
fewer than 15. Investigators use a very small dose of medication to make sure it isn’t harmful
to humans before they start using it in higher doses for later phases.

If the medication acts differently than expected, the investigators will likely to do some
additional preclinical research before deciding whether to continue the trial.

During phase I of a clinical trial, investigators spend several months looking at the effects of
the medication on about 20 to 80 people who have no underlying health conditions.

This phase aims to figure out the highest dose humans can take without serious side
effects. Investigators monitor participants very closely to see how their bodies react to
the medication during this phase.

Phase II of a clinical trial involves several hundred participants who are living with the
condition that the new medication is meant to treat. They’re usually given the same dose that
was found to be safe in the previous phase. This is also done with a placebo where a
group is given only a placebo medicine, whereas the other is given the actual
medicine.

Phase III of a clinical trial usually involves up to 3,000 participants who have the condition
that the new medication is meant to treat. Trials in this phase can last for several years.

The purpose of phase III is to evaluate how the new medication works in comparison to
existing medications for the same condition. To move forward with the trial, investigators need
to demonstrate that the medication is at least as safe and effective as existing treatment
options.

To do this, investigators use a process called randomization. This involves randomly


choosing some participants to receive the new medication and others to receive an

www.insightsactivelearn.com 1
Total Marks : 200
TEST - 3 (Textbook)
( INSTA Prelims Test Series 2021 )

existing medication.
Phase III trials are usually double-blind, which means that neither the participant nor
the investigator knows which medication the participant is taking. This helps to
eliminate bias when interpreting results.

Phase IV clinical trials happen after the FDA has approved medication. This phase involves
thousands of participants and can last for many years.

Q Source: Frequently in news

2. Consider the following about the Nitrogen cycle on earth.


1. The atmospheric nitrogen cannot be utilized as a nutrient directly by plants.
2. Blue-green algae present in the soil fix nitrogen from the atmosphere and convert it into
compounds of nitrogen.
3. Lightning fixes atmospheric nitrogen.

Select the correct answer using the codes below.


A. 1, 2 and 3
B. 1 and 3 only
C. 1 and 2 only
D. 2 and 3 only

Correct Answer : A

Answer Justification :

Justification: Nitrogen is one of the essential constituents of all living organisms as part of
proteins, chlorophyll, nucleic acids and vitamins.

Once nitrogen is converted into these usable compounds, it can be utilised by plants
from the soil through their root system.
Nitrogen is then used for the synthesis of plant proteins and other compounds. Animals
feeding on plants get these proteins and other nitrogen compounds.

Statement 2: Certain bacteria and blue green algae present in the soil fix nitrogen from the
atmosphere and convert into compounds of nitrogen

When plants and animals die, bacteria and fungi present in the soil convert the nitrogenous
wastes into nitrogenous compounds to be used by plants again. Certain other bacteria convert
some part of them to nitrogen gas which goes back into the atmosphere.

As a result, the percentage of nitrogen in the atmosphere remains more or less constant.

www.insightsactivelearn.com 2
Total Marks : 200
TEST - 3 (Textbook)
( INSTA Prelims Test Series 2021 )

Statement 3: Lightning also fixes atmospheric nitrogen.

Image credit: Studyacs.com

Q Source: Page 27: Fig. 2.10: Nitrogen cycle: Science 8th NCERT

3. Which of these important territories, recently seen in news, border the Mediterranean Sea?
1. West Bank
2. Gaza strip
3. Jerusalem

Select the correct answer using the codes below.


A. 1 and 2 only
B. 2 only
C. 3 only
D. 1, 2 and 3

Correct Answer : B

Answer Justification :

Justification: See map below.

Israel-UAE have arrived at an important peace agreement that has the potential to change the
geopolitics of West Asia and beyond.

Under the Deal the UAE will establish diplomatic relations with Israel in return for
President Binyamin Netanyahu committing to giving up a stated plan to annex the West
www.insightsactivelearn.com 3
Total Marks : 200
TEST - 3 (Textbook)
( INSTA Prelims Test Series 2021 )

Bank.
West bank is the main territory of a state that the Palestinians want.
The UAE becomes the first Gulf Arab country and third Arab nation to recognize Israel
after Egypt (in 1979) and Jordan (1994).
This deal would dramatically bring all Sunni nations in the region in an anti-Iran alliance
with Israel.

Q Source: Insights current events

4. Consider a half-filled cylindrical container X1 with total capacity V (in litres). If you were to transfer the liquid
in X1 to another cylindrical container X2, with the same base radius, but exactly half the height of X1, you will
note that the

A. Pressure at the bottom of the vessel X2 is greater than that of X1.


B. Pressure at the bottom of the vessel X2 is lesser than that of X1.
C. Pressure at the bottom of the vessel X2 is equal to that of X1.

www.insightsactivelearn.com 4
Total Marks : 200
TEST - 3 (Textbook)
( INSTA Prelims Test Series 2021 )

D. No conclusion can be made.

Correct Answer : C

Answer Justification :

Justification: The force acting on a unit area of a surface is called pressure.

The total pressure exerted by a liquid column at its base is proportional to the height of the
column.

The volume of a cylinder is = Area of base (A) X height (h). Weight of a liquid is proportional
to its mass (or volume here).

Since, area of both cylinders is the same and weight of liquid is same as well (i.e. force acting
due to the liquid), pressure applied by liquid stored in either X1 and X2 will only depend on the
height of the column. Even though the height of the container X2 is half than that of X1, the
height of liquid column will be the same in both containers, so the pressure will be same in
both as well. Answer will be C.

Q Source: Page 141: Science 8th NCERT

5. Solar flares are known to disrupt with earth’s


1. Radio communication
2. Global Positioning System (GPS)
3. Power grids
4. Satellites (artificial)

Select the correct answer using the codes below.


A. 1 only
B. 1 and 3 only
C. 1, 2, 3 and 4
D. 2 and 4 only

Correct Answer : C

Answer Justification :

Justification: Solar flares are the result of changes in magnetic fields on the sunspots that
cause a huge explosion.

These eruptions may lead to solar flares and storms.

www.insightsactivelearn.com 5
Total Marks : 200
TEST - 3 (Textbook)
( INSTA Prelims Test Series 2021 )

This phenomenon is called Coronal Mass Ejections (CME).


These solar flares are often released into space and its radiation can disrupt with earth’s
radio communication, GPS, Power grids and satellite.
Solar flares caused by these CMEs can also trigger intense light in the sky, called
auroras.
Recently, scientists developed a new model that can successfully predict seven of the
Sun’s biggest flares from the last solar cycle, out of a set of nine with the help of NASA’s
Solar Dynamics Observatory.

Q Source: Insights current events

6. Consider the following about debt instruments.


1. A debt instrument is a tool used to raise capital from the market.
2. No collateral can be demanded against a debt instrument.
3. RBI recently permitted banks to invest in debt instruments through mutual funds (MFs).

Select the correct answer using the codes below.


A. 1 and 2 only
B. 1 and 3 only
C. 1, 2 and 3
D. 3 only

Correct Answer : B

Answer Justification :

Justification: A debt instrument is a tool for an entity can utilize to raise capital.

It is a documented, binding obligation that provides funds to an entity in return for a


promise from the entity to repay a lender or investor in accordance with terms of a
contract.
Debt instrument contracts include detailed provisions on the deal such as
collateral involved, the rate of interest, the schedule for interest payments, and the
timeframe to maturity if applicable.
As per RBI’s extant Basel III guidelines, if a bank holds a debt instrument directly, it
would have to allocate lower capital as compared to holding the same debt instrument
through a mutual fund (MF)/exchange traded fund (ETF) [we will cover the concept of
ETFs in upcoming tests, as for MFs, they are a pool of money gathered from various
investors investing assets such as gold, stocks or bonds, and usually managed by a
professional investor).
RBI recently permitted banks to invest in debt instruments through mutual funds (MFs)
or exchange traded funds without allocating any additional charges.
This is to expand the bond market.

www.insightsactivelearn.com 6
Total Marks : 200
TEST - 3 (Textbook)
( INSTA Prelims Test Series 2021 )

This will result in substantial capital savings for banks and is expected to give a boost to
the corporate bond market.

Q Source:
https://www.thehindu.com/business/Industry/nod-for-banks-to-invest-in-debt-via-mfs/article322
89388.ece

7. Mixing of nutrients and oxygen in between water layers is largely dependent on

A. temperature and density of these layers


B. depth of these layers
C. relative load of aquatic population
D. None of the above

Correct Answer : A

Answer Justification :

Justification: Mixing of nutrients and oxygen in between water layers is very much
dependent on water temperatures and solute concentration (density). High temperature at the
top layer prevents proper mixing of water.

This can lead to increased bacteria levels when there is ample food supply. Many aquatic
species will fail to reproduce at elevated temperatures.

Primary producers (e.g. plants, cyanobacteria) are affected by warm water because higher
water temperature increases plant growth rates, resulting in a shorter lifespan and species
overpopulation. This can cause an algae bloom which reduces oxygen levels.

Learning: Mixing of the Surface and Deep Oceans:

Because the surface and deep ocean layers are of very different densities (due to salt content
and temperature), these layers of the ocean do not mix easily. The resistance of two water
bodies with different densities to mix is called stratification. This is a natural phenomenon of
our oceans, and can even occur in lakes. Winds and storms, such as hurricanes and typhoons,
are the mechanisms by which the surface ocean is mixed with the deep ocean. On average, it
takes the waters in the ocean’s mixed layer and deep ocean about 1,000 years (or more) to
completely mix.

Q Source: AR: Page 249: Science 8th NCERT

8. Which among the following nations in the Middle-east is/are land locked?

www.insightsactivelearn.com 7
Total Marks : 200
TEST - 3 (Textbook)
( INSTA Prelims Test Series 2021 )

1. Iraq
2. Iran
3. Syria
4. Turkey

Select the correct answer using the codes below.


A. 1, 2 and 4 only
B. 1 and 3 only
C. 2 and 4 only
D. None of the above

Correct Answer : D

Answer Justification :

Justification:

Q Source: Map-based: West Asia

www.insightsactivelearn.com 8
Total Marks : 200
TEST - 3 (Textbook)
( INSTA Prelims Test Series 2021 )

9. Consider the following about Diptheria, a disease that is common amongst children in India.
1. It is caused by a bacterium.
2. It only affects the respiratory system.
3. Diptheria is included in India’s Universal Vaccination Program (Indradhanush).

Select the correct answer using the codes below.


A. 1 only
B. 1 and 3 only
C. 2 only
D. 1, 2 and 3

Correct Answer : B

Answer Justification :

Justification: Diphtheria is caused by a bacterium now known as Corynebacterium


diphtheriae . The disease would start out as an infection of the respiratory tract and the
bacterium would secrete a toxin that injured and then destroyed cells. A thick grey substance,
from the cellular waste secreted from battling the bacterium, would envelop the pharynx and
stick to the tissues and obstruct breathing. The effects of this could travel as far the heart and
kidneys.

Through UIP, Government of India is providing vaccination free of cost


against vaccine preventable diseases include diphtheria, pertussis, tetanus, polio, measles,
severe form of childhood tuberculosis, hepatitis B, meningitis and pneumonia (Hemophilus
influenza type B infections), and Japanese encephalitis (JE).

Q Source:
https://www.thehindu.com/todays-paper/tp-features/tp-sundaymagazine/plasma-cure-or-chimer
a/article32365464.ece

10. Bhoodan Pochampally in Andhra Pradesh is famous for this double ikat woven Sari

A. Kasavu
B. Patola
C. Neriyathum
D. Bomkai

Correct Answer : B

Answer Justification :

Learning: Options A to D are all different kinds of Sari from different states of India, for e.g.

www.insightsactivelearn.com 9
Total Marks : 200
TEST - 3 (Textbook)
( INSTA Prelims Test Series 2021 )

Bokai is worn in Odisha, Kasavu and Neriyathum in Kerala.

Patola saris are a double ikat woven sari, usually made from silk, made in Patan, Gujarat.

They are very expensive, once worn only by royalty and the aristocracy, they are popular and
in demand from the affluent.

They are so unique that they are given the geographical indication tags for Gujarat as Patan
Patolas and Andhra Pradesh as Bhoodan Pochampally.

Q Source: Page 80: History Standard 8th NCERT: Our pasts: Part II

11. Which of the following missiles or missile systems have been indigenously developed in India?
1. HAMMER
2. SPICE
3. MICA

Select the correct answer using the codes below.


A. 2 only
B. 1 and 3 only
C. 1, 2 and 3
D. None of the above

Correct Answer : D

Answer Justification :

Justification: Highly Agile Modular Munition Extended Range Missiles (HAMMER) is a

www.insightsactivelearn.com 10
Total Marks : 200
TEST - 3 (Textbook)
( INSTA Prelims Test Series 2021 )

medium-range modular air-to-ground weapon designed for the French Air Force and the Navy.

It a rocket-enabled precision missile with a range of 60 km perfectly suited for high


altitude.
India has decided to fit HAMMER missiles on the newly-acquired Rafale jet aircraft
Earlier Rafale jets with HAMMER missiles carried out airstrikes in Libya, Afghanistan,
Iraq, and Syria.
Apart from the HAMMER missiles, the Rafale aircraft will also be armed with beyond-
visual range missiles like Meteor, SCALP, and MICA, increasing their ability to take on
incoming targets from a distance.

Munitions and their origins

HAMMER - Highly Agile Modular Munition Extended Range Missile – Made in France
MICA - Fire and Forget short and Medium-Range Missile System – Made in France
Meteor - Radar guided beyond-visual-range air-to-air missile – Made in France
SCALP - Beyond visual range air to air missile – Made in France
SPICE - Smart, Precise Impact, Cost-Effective Air-to-Surface missile – Made in Israel
Storm Shadow - General Purpose Long Range Cruise Missile – Made in UK

Q Source:
https://www.hindustantimes.com/india-news/india-to-boost-rafale-capabilities-with-hammer-mis
siles-under-emergency-order-report/story-Xf0SGVcipUgXvL2qRU3FZM.html

12. Consider the following about Ammonium nitrate.


1. It is a highly combustible/inflammatory material when burnt on its own even at low
temperatures.
2. It is widely used as a fertilizer.
3. It produces carbon dioxide and water vapour as end products of combustion.

Select the correct answer using the codes below.


A. 2 only
B. 1 and 2 only
C. 1, 2 and 3
D. 1 and 3 only

Correct Answer : A

Answer Justification :

Justification: Ammonium nitrate (NH₄NO₃) is one of the world’s most widely used fertilizers.

It is produced as small porous pellets, or ‘prills’,

www.insightsactivelearn.com 11
Total Marks : 200
TEST - 3 (Textbook)
( INSTA Prelims Test Series 2021 )

Ammonium nitrate does not burn on its own, instead it acts as a source of
oxygen that can accelerate the combustion (burning) of other materials.
Ammonium nitrate prills provide a much more concentrated supply of oxygen than the
air around us.
This is why it is effective in mining explosives, where it’s mixed with oil and other fuels.
At high enough temperatures, however, ammonium nitrate can violently decompose on
its own.
This process creates gases including nitrogen oxides and water vapour.
It is this rapid release of gases that causes an explosion.
Recently there was an ammonium nitrate blast in Lebanese capital Beirut, which injured
many.

Q Source: Current events

13. A disagreement between both the houses may require a joint sitting of the Parliament for which of these kinds
of bills?
1. Constitutional amendment bill
2. Money Bill
3. Ordinary Bill

Which of the above is/are correct?


A. 1 only
B. 1 and 3 only
C. 3 only
D. 2 and 3 only

Correct Answer : C

Answer Justification :

Learning: Any ordinary law needs to be passed by both the Houses. But if there is a
difference between the two Houses, the final decision is taken in a joint session in which
members of both the Houses sit together. Because of the larger number of members, the view
of the Lok Sabha is likely to prevail in such a meeting.

But, Lok Sabha exercises more powers in money matters. Once the Lok Sabha passes the
budget of the government or any other money related law, the Rajya Sabha cannot reject it.
The Rajya Sabha can only delay it by 14 days or suggest changes in it. The Lok Sabha may or
may not accept these changes. There is no provision for joint sitting in such cases.

Constitutional amendment bills need to be passed by both houses, and no house retain primacy
over the other, however joint sittings are not permitted.

Q Source: Chapter 5: Democratic Politics-I

www.insightsactivelearn.com 12
Total Marks : 200
TEST - 3 (Textbook)
( INSTA Prelims Test Series 2021 )

14. Consider the following about Hoolock Gibbons.


1. They are native to regions of South Asia, including India.
2. They are the only ape found in India.
3. They are listed as Critically endangered under the IUCN Red list.

Select the correct answer using the codes below.


A. 1 and 2 only
B. 2 only
C. 1 only
D. 1 and 3 only

Correct Answer : A

Answer Justification :

Justification: Hoolock Gibbons are native to eastern Bangladesh, Northeast India and
Southwest China.

The tailless Hoolock Gibbon is the only ape found in India.


In the areas of high tree canopies, the gibbons swing from branch to branch.
They cannot adapt to living on the ground and cannot bear the high temperatures
brought about by the loss of green cover.

The Hoolock Gibbon is categorized into two types:

1. Western hoolock gibbon - It inhibits in all the states of the north-east, restricted
between the south of the Brahmaputra River and east of the Dibang River.
It is listed as Endangered under the International Union for Conservation of Nature
(IUCN) Red List.
1. Eastern hoolock gibbon - It inhabits specific pockets of Arunachal Pradesh and Assam
in India, and in southern China and north-east Myanmar outside India.
It is listed as Vulnerable under the IUCN Red list.
In India, both the species are listed on Schedule 1 of the Indian (Wildlife) Protection Act
1972.

Q Source:
https://www.thehindu.com/sci-tech/energy-and-environment/hoolock-gibbons-feel-quite-at-hom
e-in-this-assam-village/article24993870.ece

15. How will you identify/detect an exoplanet?


1. It has a wobbly orbit.
2. It orbits a star within our solar system.

www.insightsactivelearn.com 13
Total Marks : 200
TEST - 3 (Textbook)
( INSTA Prelims Test Series 2021 )

3. It can be detected using radio waves.

Select the correct answer using the codes below.


A. 1 and 2 only
B. 2 and 3 only
C. 1 and 3 only
D. 3 only

Correct Answer : C

Answer Justification :

Justification: Discovery of Exoplanet using Radio waves

An exoplanet is a planet that orbits a star outside the solar system.


These exoplanets are hard to detect because they are hidden by the bright glare of the
stars they orbit around.
One of the key features of the exoplanet is that its orbit is wobbly because the star’s
gravitation is not at its centre which makes the phenomenon possible.
Recently, scientists have been able to discover an exoplanet and a wobbly star using just
radio waves.
In this method, scientists detect an exoplanet via auroras formed on it by the interaction
of the star and a strong magnetic field around a planetary body.

Learning: Radio waves:

Radio waves are a type of electromagnetic radiation with wavelengths in the


electromagnetic spectrum longer than infrared light.
Radio waves have frequencies as high as 300 gigahertz (GHz) to as low as 30 hertz (Hz).
The wavelength of a radio wave can be anywhere from shorter than a grain of rice to
longer than the radius of the Earth.
Like all other electromagnetic waves, radio waves travel at the speed of light in vacuum.
They are generated by electric charges undergoing acceleration, such as time varying
electric currents.
Naturally occurring radio waves are emitted by lightning and astronomical objects.
Radio waves are generated artificially by transmitters and received by radio receivers,
using antennas.

Q Source: Insights current events

16. Which of the following is NOT usually considered under the definition of “stressed assets” in the financial
sector?

www.insightsactivelearn.com 14
Total Marks : 200
TEST - 3 (Textbook)
( INSTA Prelims Test Series 2021 )

A. Non-performing assets
B. Written off assets
C. Restructured loans
D. Assets with poor rate of returns

Correct Answer : D

Answer Justification :

Justification: Assets of the banking system comprises of loans given and investment (in
bonds) made by banks. Quality of the asset indicates how much of the loans taken by the
borrowers are repaid in the form of interests and principal.

Stressed assets = NPAs + Restructured loans + Written off assets


NPA - means interest or principal is not repaid by the borrower during a specified time
period (‘overdue’ for a period of 90 days.)
Bad assets are further classified into substandard asset, doubtful asset, and loss assets
depending upon how long a loan remains as an NPA.
Restructured loans - Assets which got an extended repayment period, reduced interest
rate, converting a part of the loan into equity, providing additional financing, or some
combination of these measures.
Hence, under restructuring a bad loan is modified as a new loan.
Written off Assets - Those the bank or lender doesn’t count the money borrower owes
to it.
The financial statement of the bank will indicate that the written off loans are
compensated through some other way.

Recently RBI set up an expert committee under K V Kamath to suggest financial parameters
for resolution of covid-19 related stressed assets.

The committee will submit its recommendations to the RBI, which will notify them along
with modifications, if any, in 30 days.
Resolution Framework for Covid-19-related Stress envisages to make recommendations
on the required financial parameters to be factored in into the resolution plans, with
sector-specific benchmark.

Q Source:
https://indianexpress.com/article/business/banking-and-finance/rbi-sets-up-panel-under-k-v-ka
math-to-suggest-steps-to-deal-with-coronavirus-related-stressed-assets-6544280/

17. Consider the following about some of the major operations of Indian armed forces.

www.insightsactivelearn.com 15
Total Marks : 200
TEST - 3 (Textbook)
( INSTA Prelims Test Series 2021 )

1. Operation Meghdoot against the Pakistan Army resulted in the Indian Army getting overall
control of the Siachen Glacier in the 1980s.
2. Operation Rakshak was a border control operation conducted near Sir Creek region during the
height of insurgency in Punjab in the 1990s.

Select the correct answer using the codes below.


A. 1 only
B. 2 only
C. Both 1 and 2
D. None

Correct Answer : A

Answer Justification :

Justification: Operation Meghdoot

It was launched in the year 1984 launched in the Siachen Glacier against the Pakistan Army.
The military action resulted in the Indian Army getting overall control of the entire Siachen
Glacier. Located in the Eastern Karakoram Range, it is the second-longest glacier in the
world’s non-polar areas. It is also the highest battlefield in the world.

Operation Rakshak

It is a counter-insurgency and counter-terrorism operation started during the height of the


insurgency in Jammu and Kashmir in June 1990. This was extended to more areas in 1991.
Between 2015 and 2017, more than 750 Indian Army personnel died in this operation of the
armed forces.

Q Source: AR:
https://zeenews.india.com/india/36-years-of-operation-meghdoot-indian-army-says-doing-difficu
lt-is-routine-here-2276144.html

18. The majority of the World’s Polyethylene Terephthalate (PET) production is deployed for synthetic fibres
raising significant environmental concerns. Consider the following about it.
1. PET cannot be degraded by any species of bacteria.
2. PET resin generates toxic emissions of nickel and Benzene.

Which of the above is/are correct?


A. 1 only
B. 2 only
C. Both 1 and 2
D. None

www.insightsactivelearn.com 16
Total Marks : 200
TEST - 3 (Textbook)
( INSTA Prelims Test Series 2021 )

Correct Answer : B

Answer Justification :

Justification: It is the most common thermoplastic polymer resin of the polyester family and
is used in fibres for clothing, containers for liquids and foods, thermoforming for
manufacturing, and in combination with glass fibre for engineering resins.

At least one species of bacterium in the genus Nocardia can degrade PET with an
esterase enzyme.

Japanese scientists have isolated a bacterium Ideonella sakaiensis that possesses two enzymes
which can break down the PET into smaller pieces that the bacterium can digest.

Manufacturing PET resin generates more toxic emissions (nickel, ethyl benzene,
ethylene oxide, benzene) than manufacturing glass. Producing a 16 oz. PET bottle
generates more than 100 times the toxic emissions to air and water than making the
same size bottle out of glass.

In 2016, it was estimated that 56 million tons of PET are produced each year. While most
thermoplastics can, in principle, be recycled, PET bottle recycling is more practical than many
other plastic applications because of the high value of the resin and the almost exclusive use of
PET for widely used water and carbonated soft drink bottling.

Q Source: AR: Page 35: Science 8th NCERT

19. Global Alliance for Vaccines and Immunization (GAVI) is a/an

A. Doctors without Borders (DwB) enterprise


B. Initiative of Bill and Melinda Gates Foundation
C. World health Organization (WHO) led initiative
D. Partnership of both public and private sector organizations

Correct Answer : D

Answer Justification :

Justification: GAVI, officially Gavi, the Vaccine Alliance (previously the GAVI Alliance, and
before that the Global Alliance for Vaccines and Immunization) is a public–private global
health partnership with the goal of increasing access to immunisation in poor countries.

GAVI brings together developing country and donor governments, the World Health
Organization, UNICEF, the World Bank, the vaccine industry in both industrialised and
developing countries, research and technical agencies, civil society, the Bill & Melinda Gates
Foundation and other private philanthropists. GAVI has observer status at the World Health

www.insightsactivelearn.com 17
Total Marks : 200
TEST - 3 (Textbook)
( INSTA Prelims Test Series 2021 )

Assembly.

The Global Alliance for Vaccines and Immunizations (GAVI) is supported by India also since
our country has pledged $15 million to this and also India has shared its stocks of medicines
with over 120 countries in the wake of the pandemic.

Q Source: In news frequently

20. The term “Biosphere” subsumes


1. Lithosphere
2. Hydrosphere
3. Atmosphere

Which of the above is/are correct?


A. 1 only
B. 1 and 2 only
C. 1, 2 and 3
D. 3 only

Correct Answer : C

Answer Justification :

Justification: Biosphere is that part of the earth in which living organisms exist or which
supports life.

It is the worldwide sum of all ecosystems. It can also be termed the zone of life on Earth, a
closed system (apart from solar and cosmic radiation and heat from the interior of the Earth),
and largely self-regulating.

In other words, it is the global ecological system integrating all living beings and their
relationships, including their interaction with the elements of the lithosphere, geosphere,
hydrosphere, and atmosphere.

Q Source: Page 78: Science 8th NCERT

21. Which of the following communicable diseases can be transmitted through air?
1. Measles
2. Polio
3. Cholera

www.insightsactivelearn.com 18
Total Marks : 200
TEST - 3 (Textbook)
( INSTA Prelims Test Series 2021 )

4. Hepatitis B
5. Tuberculosis

Select the correct answer using the codes below.


A. 1, 2 and 5 only
B. 2, 3 and 5 only
C. 1, 3 and 4 only
D. 1, 2, 3, 4 and 5

Correct Answer : A

Answer Justification :

Justification: Statement 1: Measles is caused by a Virus and spreads through Air.

Statement 2: Polio caused by a Virus spreads through Air/Water.

Statement 3: Cholera and typhoid are caused by Bacteria and spread through Water. Cholera
also spreads through Food.

Statement 4: Hepatitis B is caused by a Virus and spreads through Water.

Statement 5: Tuberculosis caused by Bacteria is spread through Air. Patient should be kept in
complete isolation in such cases.

Q Source: Page 24: Science 8th NCERT

22. The Mediterranean Sea connects to the Atlantic Ocean by the

A. Suez Canal
B. Strait of Gibraltar
C. Straits of the Dardanelles
D. Straits of Bosporus

Correct Answer : B

Answer Justification :

Justification: The Mediterranean is positioned between Europe to the north, Africa to the
south, and Asia to the east.

www.insightsactivelearn.com 19
Total Marks : 200
TEST - 3 (Textbook)
( INSTA Prelims Test Series 2021 )

The Mediterranean Sea connects:

to the Atlantic Ocean through the Strait of Gibraltar (known in Homer‘s writings as the
“Pillars of Hercules“) in the west
to the Sea of Marmara and the Black Sea, by the Straits of the Dardanelles and the
Bosporus respectively, in the east
The 163 km (101 mi) long artificial Suez Canal in the southeast connects the
Mediterranean Sea to the Red Sea.

Why in News?

France has temporarily reinforced its military presence in the eastern Mediterranean Sea
amid tensions between neighbours Greece and Turkey over recently discovered gas reserves.

Q Source: Insights current events

23. Consider the following about Krishi Megh, a recent initiative.


1. It is under the aegis of the ICAR - Indian Council of Agricultural Research.
2. It has been set up under the National Agricultural Higher Education Project (NAHEP).
3. It will apply artificial intelligence and deep learning software techniques to improve
agricultural practices in India.

www.insightsactivelearn.com 20
Total Marks : 200
TEST - 3 (Textbook)
( INSTA Prelims Test Series 2021 )

Select the correct answer using the codes below.


A. 2 only
B. 3 only
C. 1, 2 and 3
D. 1 and 3 only

Correct Answer : C

Answer Justification :

Justification: Union Minister of Agriculture & Farmers’ Welfare virtually launched the Krishi
Megh (National Agricultural Research & Education System -Cloud Infrastructure and
Services).

Krishi Megh is the data recovery centre of ICAR (Indian Council of Agricultural Research).

It has been set up under the National Agricultural Higher Education Project (NAHEP).

The data recovery centre has been set up at National Academy of Agricultural Research
Management (NAARM), Hyderabad.

Significance and benefits of Krishi Megh:

Built to mitigate the risk, enhance the quality, availability and accessibility of e-
governance, research, extension and education in the field of agriculture in India.
Krishi Megh is equipped with the latest artificial intelligence and deep learning software
for building and deploying of deep learning-based applications through image analysis,
disease identification in livestock, etc.
It enables the farmers, researchers, students and policymakers to be more equipped
with the updated and latest information regarding agriculture and research.

Learning: National Agricultural Higher Education Project (NAHEP):

The project is funded by both the government of India and the World Bank.

The overall objective of the project is to provide more relevant and high-quality education to
the agricultural university students that is in tune with the New Education Policy – 2020.

Q Source:
https://www.insightsonindia.com/2020/08/13/insights-daily-current-affairs-pib-summary-13-aug
ust-2020/

24. The Pole Star may NOT appear to remain in the same position with respect to the North Pole if
1. Earth’s spherical shape is distorted, with the axis and precession remaining the same.
2. Earth’s orbital velocity increases, with its precession, shape and axial tilt remaining the same.

www.insightsactivelearn.com 21
Total Marks : 200
TEST - 3 (Textbook)
( INSTA Prelims Test Series 2021 )

Which of the above is/are correct?


A. 1 only
B. 2 only
C. Both 1 and 2
D. None

Correct Answer : A

Answer Justification :

Justification: Notice in the diagram why the pole star appears to remain at the same place
with respect to the north pole.

S1: If everything remains the same, except that the position of North pole is changed (and it is
no longer at the centre of the circle of precession due to earth’s shape distortion), you will see
that the position of pole star will keep changing from North Pole. But, it will still remain to be
the same from the new polar centre (of the distorted earth, not the previous spherical earth
which was North pole).

www.insightsactivelearn.com 22
Total Marks : 200
TEST - 3 (Textbook)
( INSTA Prelims Test Series 2021 )

S2: This is incorrect, since orbital speed has nothing to do with the relative location of the pole
star. See diagram above.

Learning: In ancient times, people used to determine directions during the night with the
help of stars.

The North star (Pole star) indicates the north direction. It always remains in the same position
in the sky.

You should see this link to understand how we can measure it

http://study.com/academy/lesson/using-the-sun-stars-to-determine-latitude-longitude.html

Latitude is basically the angle of your place from the centre of the Earth (equatorial position).
You can either find out this angle OR find out the angle of your place with respect to the North
Pole. Finding the latter is easier as Pole Star is always found vertically above the North Pole.

If you can find your place’s angle from the Pole star (North Pole), just subtract it from 90
degrees to arrive at your place’s latitude.

Q Source: Page 223: Science 8th NCERT

25. Arrange the following regions/locations in West Asia from East to West:
1. Beirut
2. Damascus
3. Baghdad
4. Tehran

Select the correct answer using the codes below.


A. 3421
B. 4312
C. 3412
D. 4321

Correct Answer : D

Answer Justification :

Justification:

www.insightsactivelearn.com 23
Total Marks : 200
TEST - 3 (Textbook)
( INSTA Prelims Test Series 2021 )

Q Source: Map based question: Region: West Asia

26. Consider the following about the SRIJAN Portal, recently seen in news.
1. It provides information regarding the defence equipments imported by the Defence Public
Sector Units (PSUs) in the country so that they are indigenized.
2. It was launched by the Department of Defence Production to boost Make in India initiatives in
the defence sector.

Select the correct answer using the codes below.


A. 1 only
B. 2 only
C. Both 1 and 2
D. None

Correct Answer : C

Answer Justification :

Justification: On the final day of the ‘Atmanirbhar Bharat Week’ celebrations of the Ministry
www.insightsactivelearn.com 24
Total Marks : 200
TEST - 3 (Textbook)
( INSTA Prelims Test Series 2021 )

of Defence from Aug 10 – 14, 2020, the Defence Minister virtually launched the Department of
Defence Production Portal ‘SRIJAN’ as a part of opportunities for Make in India initiatives in
the defence sector.

About the SRIJAN Portal

This portal provides information regarding the defence equipment and items which are being
imported currently or to be imported by the Defence PSUs in the country. The government will
take proper measures regarding making these products indigenized.

In the first stage, these items are displayed with the value of imports of Rs 1 million and
above. For 2019-20, there were 1557 such items whereas the items were reduced to 739 in
2020-21.

Learning: Defence Testing Infra Scheme (DTIS)

The Defence Ministry launched the Defence Testing Infra Scheme in May 2020 in order to
create testing infra for the defence sector in the country. The objective is to boost the
domestic defence and aerospace manufacturing in the country. The initial cost has been set at
Rs 400 crores for this initiative.

Naval Innovation and Indigenization Organization (NIIO)

This is another initiative of the Ministry of Defence in order to inculcate the values of
innovation in the defence production of the country. NIIO will be a one-stop solution through
which the stakeholders will get in touch with one another in order to foster innovation and
indigenization for self-reliance in defence, in line with the ‘Atmanirbhar Bharat’ pledge of the
Government of India. The N-TAC will administer this platform.

Q Source: Current events

27. In a sovereign country, the following necessarily follow:


1. There is freedom of speech and expression for all citizens.
2. All citizens are given equal social and economic rights.
3. The country has its own elected government.

Select the correct answer using the codes below.


A. 1 and 2 only
B. 1 only
C. 3 only
D. None of the above

Correct Answer : D

Answer Justification :

www.insightsactivelearn.com 25
Total Marks : 200
TEST - 3 (Textbook)
( INSTA Prelims Test Series 2021 )

Justification: Let’s take this in India’s context, India is a sovereign nation.

Sovereignty simply implies that India is a state that takes its own decisions ultimately guided
by the people. No external agency dictates terms to India.

However, the notion of sovereignty does not seep into the framework of all democratic rights.
For e.g. a sovereign state might very well discriminate between its citizens, for e.g. as
Pakistan does with all non-Muslims.

Same applies for freedom of speech and expression. A sovereign state may very well restrict
freedom of speech, as it is not an absolute right, even though it is vital to a democracy.

Also, sovereign states can be a monarchy or even an autocracy. Sovereignty does not imply
democracy, but vice versa may apply.

Q Source: Chapter 3: Democratic Politics - I

28. Demo-2 Mission of NASA and SpaceX is primarily a

A. Mars orbiter program


B. Commercial crew program
C. Solar space drone
D. Reusable launch vehicle development program

Correct Answer : B

Answer Justification :

Justification: The Demo-2 mission is part of NASA’s Commercial Crew Program with the aim
of developing reliable and cost-effective access to and from the International Space Station.

Crew Dragon is a part of the Dragon 2, a class of reusable spacecraft developed and
manufactured by American aerospace manufacturer SpaceX.

It is the 5th class of US spacecraft to take human beings into orbit, after the Mercury,
Gemini, Apollo and Space Shuttle programs.
Falcon 9, which carried the spaceship into the orbit, was also built by SpaceX.
It is done under the Demo-2 Mission of NASA and SpaceX.
Recently, dragon capsule returned back to earth after its short test flight, this clears the
way for possible tourist flights in the near future.
It was the first splashdown by U.S. astronauts in 45 years, with the first commercially
built and operated spacecraft to carry people to and from orbit.
The last time NASA astronauts returned from space to water was on July 24, 1975, in the
Pacific to end a joint U.S.-Soviet mission known as Apollo-Soyuz.

Q Source: Current events

www.insightsactivelearn.com 26
Total Marks : 200
TEST - 3 (Textbook)
( INSTA Prelims Test Series 2021 )

29. Indian Computer Emergency Response Team (CERT-In) performs which of these functions?
1. Allocation of Domain Name System (DNS) to servers
2. Routing cyber traffic between different networks
3. Issuing forecast and alerts of cyber security incidents

Select the correct answer using the codes below.


A. 1 and 2 only
B. 2 and 3 only
C. 1 only
D. 3 only

Correct Answer : D

Answer Justification :

Justification: Established by IT Act, 2008, It is the nodal agency to deal with cyber security
threats like hacking and phishing. It strengthens security-related defence of the Indian
Internet domain. In March 2014, CERT-In reported a critical flaw in Android Jelly bean's VPN
implementation.

It performs the following functions in the domain of cyber security:

Collection, analysis and dissemination of information on cyber incidents


Forecast and alerts of cyber security incidents
Emergency measures for handling cyber security incidents
Coordination of cyber incident response activities
Issue guidelines, advisories, vulnerability notes and whitepapers relating to information
security practices, procedures, prevention, response and reporting of cyber incidents

Q Source: Frequently in news due to incidents of Spear Phishing

30. The National Green Tribunal (NGT) has called for commercial entities to follow new rules for getting
permission to extract groundwater. Consider the following about these rules.
1. Groundwater extraction contracts will not be given for perpetuity.
2. Environmental Impact Assessment (EIA) will now form the basis for granting such permissions
to commercial entities.
3. Plants already involved in commercial extraction of ground water will undergo individual
environmental assessment through an expert committee setup by the government.

www.insightsactivelearn.com 27
Total Marks : 200
TEST - 3 (Textbook)
( INSTA Prelims Test Series 2021 )

Select the correct answer using the codes below.


A. 1 and 2 only
B. 1, 2 and 3
C. 3 only
D. 2 and 3 only

Correct Answer : B

Answer Justification :

Justification: The National Green Tribunal (NGT) has called for commercial entities to follow
new rules for getting permission to extract groundwater.

Environmental Impact Assessments (EIA) will now form the basis of granting such
permissions.
The Union Ministry of Jal Shakti and the Central Ground Water Board (CGWB) were
ordered by the NGT to ensure no general permission was given for withdrawing
groundwater, particularly to any commercial entity.
Under the order, plants involved in commercial extraction of ground water will undergo
individual assessment through an expert committee.
All overexploited, critical and semi-critical (OCS) assessment units must undergo water
mapping.
Water management plans need to be prepared for all OCS assessment units in the
country based on the mapping data, starting with overexploited blocks.
NGT had also mentioned that permission to extract groundwater must be for specified
times and quantity of water, not for perpetuity.
It must be necessarily subject to digital flow meters, which cannot be accessed by
proponents, with mandatory annual calibration by the authorized agency at proponents.
Section 5 of the Environment Protection Act that allows the Centre to shut down
industrial units that grossly violate the law.
The Union Ministry of Environment, Forests and Climate Change ordered closure of the
LG Polymers plant in Vishakapatnam, Andhra Pradesh.

Q Source: Current events

31. Which of the following describes “Thuni” most appropriately?

A. It is an in situ conservation technique of the Bondas.


B. It is an ancient form of martial arts practiced in Tamil Nadu.
C. It is the traditional craft technique of the Meitis.
D. It is a harvest festival celebrated in the North-eastern India.

www.insightsactivelearn.com 28
Total Marks : 200
TEST - 3 (Textbook)
( INSTA Prelims Test Series 2021 )

Correct Answer : D

Answer Justification :

Learning: It is one of the major festivals celebrated by the Chakhesang tribe, predominantly
by the Poumai Chakhesang from the Razeba area.

For the Poumai community in Manipur, Thuni is a premier festival of new harvest.
It is celebrated from January 5th every year and continues for several days.
Most importantly, Thuni is a festival of equity when both the rich and the poor; the old
and the young share from the same barrel of rice beer and eat the same food.

Q Source: AR: Page 11: Harvest Festivals: Science 8th NCERT

32. Protozoan infections can affect or cause disease in


1. Terrestrial plants
2. Marine organisms
3. Mammals

Select the correct answer using the codes below.


A. 1 only
B. 1 and 2 only
C. 2 and 3 only
D. 1, 2 and 3

Correct Answer : D

Answer Justification :

Justification: Protozoan infections are parasitic diseases caused by organisms formerly


classified in the Kingdom Protozoa. They are usually contracted by either an insect vector or
by contact with an infected substance or surface Protozoa mainly feed on bacteria, but they
also eat other protozoa, and sometimes fungi. Some protozoa absorb food through their cell
tissues.

Protozoan infections are responsible for diseases that affect many different types of organisms,
including plants, animals, and some marine life. Many of the most prevalent and deadly
human diseases are caused by a protozoan infection, including African Sleeping Sickness,
amoebic dysentery, and malaria.

www.insightsactivelearn.com 29
Total Marks : 200
TEST - 3 (Textbook)
( INSTA Prelims Test Series 2021 )

Most protozoa do us no harm. But there are a few that cause diseases. One type of amoeba can
live in human intestines. It feeds on red blood cells and causes a disease known as dysentery.

All protozoa digest their food in stomach-like compartments. As they digest, they make and
give off nitrogen, which is an element that plants and other higher creatures can use.

Another species of protozoa can sicken hundreds of thousands of people when it gets into the
tap water. Perhaps the best-known deadly protozoa causes malaria.

Q Source: AR: Page 17: Science 8th NCERT

33. Consider the following about lactobacillus species of bacteria.


1. They produce hydrogen peroxide which inhibits the growth and virulence of fungal pathogens.
2. They can be used for antibacterial applications.

Which of the above is/are correct?


A. 1 only
B. 2 only
C. Both 1 and 2
D. None

Correct Answer : C

Answer Justification :

Justification: Lactobacillus is a genus of Gram-positive, aerotolerant anaerobes or


microaerophilic, rod-shaped, non-spore-forming bacteria. They are a major part of the lactic
acid bacteria group (i.e., they convert sugars to lactic acid). In humans, they constitute a
significant component of the microbiota at a number of body sites, such as the digestive
system.

The antibacterial and antifungal activity of Lactobacillus species relies on production of


bacteriocins and low molecular weight compounds that inhibits these microorganisms.

For e.g. Lactobacillus species produce hydrogen peroxide which inhibits the growth and
virulence of the fungal pathogen Candida albicans in vitro and in vivo.

Some Lactobacillus species are also used as starter cultures in industry for controlled
fermentation in the production of yogurt, cheese, sauerkraut, pickles, beer, cider, kimchi,
cocoa, kefir, and other fermented foods, as well as animal feeds.

Q Source: AR: Page 19: Science 8th NCERT

www.insightsactivelearn.com 30
Total Marks : 200
TEST - 3 (Textbook)
( INSTA Prelims Test Series 2021 )

34. Consider the following about some defence sector related initiatives.
1. i-DEX initiative intends to promote innovation and modernization in the defence industry in the
country.
2. SWAVLAMBAN aims at improving efforts of indigenization in the defence sector.

Select the correct answer using the codes below.


A. 1 only
B. 2 only
C. Both 1 and 2
D. None

Correct Answer : C

Answer Justification :

Justification: SWAVLAMBAN

The Indian Navy has published an indigenization perspective so that its plans and objectives
can be highlighted and also intimated to the public. This is named as ‘SWAVLAMBAN’ and it
has been launched during the event of launching the NIIO by the Government of India.

i-DEX Initiative

This is an initiative taken by the government in order to promote innovation and modernization
in the defence industry in the country. This initiative is aimed at encouraging innovation and
technology development in Defence and Aerospace by engaging industries, including the
MSMEs, start-ups and R&D institutes. This i-DEX fund is managed by Defence Innovation
Organization (DIO) and it also works as an executive arm of the DIO.

Learning: Organizations:

A Technology Development Acceleration Cell has also been created for induction of
emerging disruptive technology in a speedy manner.
This comes in the backdrop of the publication of the Draft Defence Acquisition Policy
which has focused on research and development in the country so that the import
dependence for the country can be reduced in the coming years.
Indian Navy already has a functional Directorate of Indigenization and the new
structures created will work with the existing one only and also on the ongoing projects
apart from emphasizing on innovation in the upcoming projects.

Q Source: In news

35. Consider the following about Sickle Cell Diseases.

www.insightsactivelearn.com 31
Total Marks : 200
TEST - 3 (Textbook)
( INSTA Prelims Test Series 2021 )

1. It affects the level of haemoglobin in the Red blood cells.


2. The disease is genetic in nature and presently incurable.
3. The sickle gene is widespread among many tribal population groups in India.

Select the correct answer using the codes below.


A. 1 and 3 only
B. 1 only
C. 2 and 3 only
D. 1, 2 and 3

Correct Answer : A

Answer Justification :

Justification: Sickle cell disease is an inherited blood condition which is most


common among people of African, Arabian and Indian origin. In disease of African
origin, research has led to models of care which prevent serious complications, improve the
quality of life, and increase survival. In India, the disease is largely undocumented. Thus, there
is an urgent need to document the features of Indian disease so that locally appropriate
models of care may be evolved.

First described in the Nilgiri Hills of northern Tamil Nadu in 1952, the sickle cell gene is now
known to be widespread among people of the Deccan plateau of central India with a smaller
focus in the north of Kerala and Tamil Nadu. Extensive studies performed by the
Anthropological Survey of India have documented the distribution and frequency of the
sickle cell trait which reaches levels as high as 35 per cent in some communities,
such as tribal communities.

Sickle cell disease is a group of disorders that affects haemoglobin, the molecule in red blood
cells that delivers oxygen to cells throughout the body. People with this disease have atypical
haemoglobin molecules called haemoglobin S, which can distort red blood cells into a sickle,
or crescent, shape.

Signs and symptoms of sickle cell disease usually begin in early childhood. Characteristic
features of this disorder include a low number of red blood cells (anaemia), repeated
infections, and periodic episodes of pain. The severity of symptoms varies from person to
person. Some people have mild symptoms, while others are frequently hospitalized for more
serious complications.

The signs and symptoms of sickle cell disease are caused by the sickling of red blood cells.
When red blood cells sickle, they break down prematurely, which can lead to anaemia.
Anaemia can cause shortness of breath, fatigue, and delayed growth and development in
children.

The disease is curable with bone marrow transplant, stem cell therapy etc.

Q Source:

www.insightsactivelearn.com 32
Total Marks : 200
TEST - 3 (Textbook)
( INSTA Prelims Test Series 2021 )

https://timesofindia.indiatimes.com/india/govt-to-build-database-on-sickle-cell-disease-ijmr-war
ns-children-with-scd-at-risk-of-covid-19-related-complications/articleshow/77584592.cms

36. Hydrogen is commercially used in


1. Hydrocracking in petroleum industry
2. Production of ammonia
3. Fuel cells for electricity generation

Select the correct answer using the codes below.


A. 1 and 2 only
B. 2 and 3 only
C. 1 only
D. 1, 2 and 3

Correct Answer : D

Answer Justification :

Justification & Learning: Some see hydrogen gas as the clean fuel of the future – generated
from water and returning to water when it is oxidised. Hydrogen-powered fuel cells are
increasingly being seen as ‘pollution-free’ sources of energy and are now being used in
some buses and cars.

Hydrogen also has many other uses. In the chemical industry it is used to make ammonia
for agricultural fertiliser (the Haber process) and cyclohexane and methanol, which are
intermediates in the production of plastics and pharmaceuticals. It is also used to remove
sulfur from fuels during the oil-refining process. Large quantities of hydrogen are used to
hydrogenate oils to form fats, for example to make margarine.

In the glass industry hydrogen is used as a protective atmosphere for making flat glass sheets.
In the electronics industry it is used as a flushing gas during the manufacture of silicon chips.

The low density of hydrogen made it a natural choice for one of its first practical uses – filling
balloons and airships. However, it reacts vigorously with oxygen (to form water) and its future
in filling airships ended when the Hindenburg airship caught fire.

Hydrogen is also used for the conversion of heavy petroleum fractions into lighter ones via the
process of hydrocracking.

Q Source: AR: Page 60: Science 8th NCERT

37. Global warming can cause low lying coastal areas to be permanently submerged due to an increase in sea water
level which can be caused by

www.insightsactivelearn.com 33
Total Marks : 200
TEST - 3 (Textbook)
( INSTA Prelims Test Series 2021 )

1. Thermal expansion of sea water


2. Increase in sea water volume due to melting of glacier

Which of the above is/are correct?


A. 1 only
B. 2 only
C. Both 1 and 2
D. None

Correct Answer : C

Answer Justification :

Justification: Global warming is the rise in temperature of the atmosphere of the earth.

This results, among other things, in the melting of polar glaciers, which leads to a rise in the
sea level, causing floods in the coastal areas (for e.g. Maldives is a low lying nation and at a
threat of being drowned by the sea level rise).Global warming has also led to massive glacial
retreat in the past 50 years.

Thermal expansion is caused when seawater expands because of the higher temperature of
the water. Since the oceans absorb heat from the atmosphere, when the atmosphere becomes
warmer so will the oceans. Warm seawater has a greater volume than cold seawater.

Q Source: Page 73: Science 8th NCERT

38. “Empowering Tribal, Transforming India” online platform, launched by Ministry of Tribal Affairs recently,
maps which of these details concerning tribal welfare schemes?
1. Scholarship Schemes
2. NGO funding
3. Land per community

Select the correct answer using the codes below.


A. 1 and 2 only
B. 1 only
C. 3 only
D. 1, 2 and 3

Correct Answer : A

Answer Justification :

www.insightsactivelearn.com 34
Total Marks : 200
TEST - 3 (Textbook)
( INSTA Prelims Test Series 2021 )

Justification: “Empowering Tribal, Transforming India” is an online performance Dashboard


launched by M/o Tribal Affairs.

It has been developed by Centre of Excellence of Data Analytics (CEDA), organization


under National Informatics Centre (NIC).
Performance Dashboard is an interactive and dynamic online platform that showcases
updated & real-time details of 11 schemes / initiatives of the Ministry for achieving
Sustainable Developmental Goals.

Highlights of the dash board are:

1. It captures performance of 5 Scholarship Schemes of Ministry wherein every year about


30 lakh underprivileged ST beneficiaries reap benefits to the tune of INR 2500 crores.
2. It also displays the details of functional schools under Eklavya Model Residential Schools
(EMRS) scheme.
3. It maps district wise NGO details, funds given to NGO and beneficiaries’ details.
Ministry of Tribal Affairs has recently got 66th SKOCH Gold Award for “Empowerment
of Tribal through IT enabled Scholarship Schemes” under the guidance of DBT Mission.
It does not contain details of tribal landholdings.

Q Source:
https://timesofindia.indiatimes.com/india/govt-to-build-database-on-sickle-cell-disease-ijmr-war
ns-children-with-scd-at-risk-of-covid-19-related-complications/articleshow/77584592.cms

39. You are likely to find which of these tree species in tropical deciduous forests of Central India?
1. Rosewood
2. Teak
3. Silver ferns
4. Sal

Which of the above is/are correct?


A. 2 and 4 only
B. 1, 2, 3 and 4
C. 2, 3 and 4 only
D. 1 and 3 only

Correct Answer : C

Answer Justification :

Learning: Tropical deciduous forests occur in regions with heavy rainfall for part of the year
followed by a marked dry season. These forest formations are dense and lush during the wet
summers, but become a dry landscape during the dry winters when most trees shed their

www.insightsactivelearn.com 35
Total Marks : 200
TEST - 3 (Textbook)
( INSTA Prelims Test Series 2021 )

leaves.

Sal, teak, mango, jamun, silver ferns, arjun, etc are the flora and chinkara, blue-bull, barking
deer, cheetal, leopard, wild dog, wolf, etc. are examples of the fauna of the Pachmarhi
Biosphere Reserve (deciduous vegetation). The biosphere reserve is located in Madhya
Pradesh.

Rosewood is a premium timber species occurring in evergreen forests, so 1 is incorrect.

Q Source: Page 80: Science 8th NCERT

40. Consider the following about the Sinai Peninsula.


1. It lies between the Gulf of Suez and Gulf of Aden.
2. It forms a land bridge between Asia and Africa.
3. Red Sea borders the Peninsula.

Which of the above is/are correct?


A. 1 and 2 only
B. 2 and 3 only
C. 1, 2 and 3
D. 3 only

Correct Answer : B

Answer Justification :

Learning: It is situated between the Mediterranean Sea to the north and the Red Sea to the
south, serving as a land bridge between Asia and Africa. It is the only part of Egyptian
territory located in Asia.

Israel invaded and occupied Sinai during the Suez Crisis.

As a result of the Israel-Egypt Peace Treaty of 1979 and subsequent efforts, Israel withdrew
from all of the Sinai Peninsula.

www.insightsactivelearn.com 36
Total Marks : 200
TEST - 3 (Textbook)
( INSTA Prelims Test Series 2021 )

Q Source: Map based question: Region: South Asia

41. If a species is endemic to Panchmarhi biosphere reserve, it implies that it is

A. an exotic species of the reserve


B. indigenous to the reserve and less likely to be found in most other regions
C. beneficial to the reserve environment
D. resides in the ecologically rich regions of the reserve

Correct Answer : B

Answer Justification :

Justification: Endemic species are not naturally found anywhere else other than the areas
where they are endemic. A particular type of animal or plant may be endemic to a zone, a state
or a country.

For e.g. Sal and Wild mango are two endemic species of Panchmarhi biosphere reserve.

Bison, Indian giant squirrel and flying squirrel are endemic fauna of this area.

You should note that a species can be endemic to more than one area, for e.g. Olive Ridley Sea
Turtles are endemic to Pacific, Atlantic and Indian oceans.

www.insightsactivelearn.com 37
Total Marks : 200
TEST - 3 (Textbook)
( INSTA Prelims Test Series 2021 )

Q Source: Page 81: Science 8th NCERT

42. IUCN publishes a Red List of which of the following?


1. Endangered plants
2. Endangered animals
3. Fragile ecosystems
4. Protected areas

Which of the above is/are correct?


A. 1 and 2 only
B. 1, 2 and 3 only
C. 1, 2, 3 and 4
D. 3 only

Correct Answer : B

Answer Justification :

Learning: The IUCN Red List of Threatened Species founded in 1964, is the world's most
comprehensive inventory of the global conservation status of biological species.

The Red List of Ecosystems is developed and implemented jointly by the IUCN Commission on
Ecosystem Management (CEM) and the IUCN Ecosystem Management Programme (EMP), in
collaboration with the IUCN Species Survival Commission (SSC) and the IUCN Global Species
Programme (GSP). They collaborate in building and promoting the Red List of Ecosystems at
global, regional and national levels. This involves engaging with partners on the ground, and
awareness-raising among policy-makers.

See it here
https://www.iucn.org/theme/ecosystem-management/our-work-gemp/red-list-ecosystems

There is no list of protected areas as explicitly maintained under an IUCN red list.

Q Source: AR: Page 83: Science 8th NCERT

43. Consider the following about the Boreal Summer Intra-Seasonal Oscillation (BSISO).
1. It brings short-term climate variability in the global monsoon system.
2. It has been associated with a prolonged relative weakening of wave activity or intensity in the
north Indian Ocean and the Arabian Sea.

Select the correct answer using the codes below.

www.insightsactivelearn.com 38
Total Marks : 200
TEST - 3 (Textbook)
( INSTA Prelims Test Series 2021 )

A. 1 only
B. 2 only
C. Both 1 and 2
D. None

Correct Answer : A

Answer Justification :

Justification: BSISO is a movement of convection (heat) from the Indian Ocean to the western
Pacific Ocean roughly every 10-50 days during the monsoon (June-September).

The BSISO of the Asian summer monsoon (ASM) is one of the most prominent sources of
short-term climate variability in the global monsoon system.
Researchers at the Indian National Centre for Ocean Information Services (INCOIS),
Hyderabad have reportedly found a way to better forecast the Boreal Summer Intra-
Seasonal Oscillation (BSISO).
They found that waves induced by active phases of BSISO are nearly 0.5 meters higher
than those which occur during other phases of BSISO.
The active phase (between June and August) enhances monsoon winds and hence the
surface waves.
Some other phases induce high wave activity in the north Indian Ocean and the Arabian
Sea
Compared with the related Madden-Julian Oscillation (MJO) it is more complex in nature,
with prominent northward propagation and variability extending much further from the
equator.
Wave forecast advisories based on the BSISO would be more useful for efficient coastal
and marine management.

Q Source:
https://www.thehindu.com/news/cities/Hyderabad/climate-forecast-system-for-surface-waves/a
rticle32271202.ece

44. The “Blind spot” in a human eye can be found at

A. both ends of the Cornea


B. junction of the optic nerve and the retina
C. end of the Ciliary muscles
D. centre of the eye Lens

Correct Answer : B

Answer Justification :

www.insightsactivelearn.com 39
Total Marks : 200
TEST - 3 (Textbook)
( INSTA Prelims Test Series 2021 )

Learning: Blind spot is a small portion of the visual field of each eye that corresponds to the
position of the optic disk (also known as the optic nerve head) within the retina. There are no
photoreceptors (i.e., rods or cones) in the optic disk, and, therefore, there is no image
detection in this area.

In other words, the eye lens focuses light on the back of the eye, on a layer called retina.
Retina contains several nerve cells. Sensations felt by the nerve cells are then transmitted to
the brain through the optic nerve. At the junction of the optic nerve and the retina, there are
no sensory cells, so no vision is possible at that spot. This is called the blind spot.

Q Source: Page 208: Science 8th NCERT

45. These planets in our Solar system have no satellites/moons of their own?
1. Mercury
2. Venus
3. Mars

Select the correct answer using the codes below.


A. 1 and 2 only
B. 2 and 3 only
C. 3 only
D. 1, 2 and 3

Correct Answer : A

www.insightsactivelearn.com 40
Total Marks : 200
TEST - 3 (Textbook)
( INSTA Prelims Test Series 2021 )

Answer Justification :

Justification: Venus and Mercury have no moon or satellites of their own. But, Mars has two
satellites.

Mars has two small satellites, Phobos and Deimos (fear and terror), which were discovered by
the American astronomer Asaph Hall in 1877.

Venus appears in the eastern sky before sunrise. Sometimes it appears in the western sky just
after sunset. Therefore it is often called a morning or an evening star, although it is not a star.

Q Source: Page 229-231: Science 8th NCERT

46. The bright steak of a meteor lasts for a very short time because

A. The friction due to the atmosphere evaporates the meteor quickly.


B. The tail of a meteor is always directed away from the sun.
C. It contains icy debris.
D. The length of the meteor’s tail grows in size as it approaches the sun.

Correct Answer : A

Answer Justification :

Justification: A meteor is a streak of light in the sky caused by a meteoroid crashing through
Earth’s atmosphere.

Meteoroids are lumps of rock or iron that orbit the sun. Most meteoroids are small fragments
of rock created by asteroid collisions. Comets also create meteoroids as they orbit the sun
and shed dust and debris.

When a meteoroid enters the Earth’s upper atmosphere, it heats up due to friction from the
air. The heat causes gases around the meteoroid to glow brightly, and a meteor appears.
Meteors are often referred to as shooting stars or falling stars because of the bright tail of
light they create as they pass through the sky. Most meteors occur in Earth’s mesosphere,
about 50-80 kilometers (31-50 miles) above the Earth's surface.

The faster and larger the meteor, the brighter and longer it may glow. The smallest meteors
only glow for about a second while larger and faster meteors can be visible for up to several
minutes. Although thousands of meteors fall during the day, meteors are best observed at
night, when the streaks of light are visible in the dark sky.

Q Source: Page 233: Science 8th NCERT

www.insightsactivelearn.com 41
Total Marks : 200
TEST - 3 (Textbook)
( INSTA Prelims Test Series 2021 )

47. Driving from Rishikesh to Patna along the banks of river Ganga, you are likely to cross which of these
places?
1. Kanpur
2. Allahabad
3. Varanasi

Which of the above is/are correct?


A. 1 only
B. 2 and 3 only
C. 3 only
D. 1, 2 and 3

Correct Answer : D

Answer Justification :

Justification: The map below shows these locations along the river banks of Ganga.

Q Source: Map-based question: Page 247: Science 8th NCERT

48. Which of the following materials is/are being used as electro catalysts for the hydrogen (H2) evolution
reaction (HER)?
1. Carbon
2. Palladium
3. Platinum

www.insightsactivelearn.com 42
Total Marks : 200
TEST - 3 (Textbook)
( INSTA Prelims Test Series 2021 )

Select the correct answer using the codes below.


A. 1 only
B. 2 only
C. 1 and 3 only
D. 1, 2 and 3

Correct Answer : D

Answer Justification :

Justification: The hydrogen evolution reaction (HER, 2 H+ + 2 e− → H2) is the cathodic


reaction in electrochemical water splitting. This is because Hydrogen is projected as one of
the next generation low carbon fuels.

The future of use of hydrogen as a fuel lies in the design of efficient electro catalysts for
facilitating electrochemical splitting of water to produce hydrogen.
The effectiveness of the electro catalyst for the hydrogen (H2) evolution reaction (HER)
largely depends on its ability to lower the potential of an electrochemical reaction
maximally, and cost of synthesis (production).
The commercially used Platinum (Pt) / Carbon (C) catalysts are efficient but expensive
and suffer from metal ion leaching or electro catalyst corrosion when used for long
duration.
Metal-organic frameworks (MOFs) and coordination polymers (COPs) are envisioned as
the next generation catalysts.
Centre for Nano and Soft Matter Sciences (CeNS) an autonomous institute under
Department of Science and Technology, have synthesized a novel COP consisting of
palladium Pd(II) ions.
The recent invention serve as a source of active sites for H-adsorption, and benzene
tetra mine (BTA) chelating ligands capable of better charge transfer.

Q Source:
https://www.thehindu.com/sci-tech/energy-and-environment/on-the-threshold-of-a-hydrogen-re
volution/article26903374.ece

49. Red Sea borders which of these nations?

A. Turkey and Syria


B. Israel and Syria
C. Jordan and Saudi Arabia
D. Lebanon and Israel

Correct Answer : C

www.insightsactivelearn.com 43
Total Marks : 200
TEST - 3 (Textbook)
( INSTA Prelims Test Series 2021 )

Answer Justification :

Q Source: Map based question: Region: West Asia

50. Objectives of the Wood’s Despatch 1854 included


1. Promoting studies in vernacular languages and ancient Indian texts
2. Developing practical and vocational skills of the Indians
3. Raising the moral character of the young generation
4. Privatizing education system in India

Select the correct answer using the codes below.


A. 2 and 3 only
B. 1, 3 and 4 only
C. 2 only
D. 1, 2, 3 and 4

Correct Answer : A

Answer Justification :
www.insightsactivelearn.com 44
Total Marks : 200
TEST - 3 (Textbook)
( INSTA Prelims Test Series 2021 )

Justification: The Despatch had the following objectives in view:

To impart Western knowledge, information about the western culture to the Indians.
To educate the natives of India so that a class of public servants could be created.
To promote intellectual development and also raise the moral character of the young
generation.
To develop practical and vocational skills of the Indians people so that more and more
articles could be produced and also to create a good market for consumption of those
goods.

The Wood’s Despatch recommended the sanction of a grant-in-aid system in the


Indian educational system and argued that schools must charge fees from their
students.

Outlining the educational policy that was to be followed in India, it emphasised once again the
practical benefits of a system of European learning, as opposed to Oriental One of the practical
uses the Despatch pointed to was economic.

European learning, it said, would enable Indians to recognise the advantages that flow
from the expansion of trade and commerce, and make them see the importance of
developing the resources of the country.
Introducing them to European ways of life, would change their tastes and desires, and
create a demand for British goods, for Indians would begin to appreciate and buy things
that were produced in Europe.
Following the 1854 Despatch, several measures were introduced by the British.
Education departments of the government were set up to extend control over all matters
regarding education. Steps were taken to establish a system of university education

Q Source: AR: Page 98: History Standard 8th NCERT: Our pasts: Part II

51. Coal gas or Syngas is a mixture consisting primarily of


1. Methane (CH4)
2. Carbon monoxide (CO)
3. Ammonia (NH3)

Which of the above is/are correct?


A. 1 only
B. 2 and 3 only
C. 2 only
D. 1 and 2 only

Correct Answer : D

www.insightsactivelearn.com 45
Total Marks : 200
TEST - 3 (Textbook)
( INSTA Prelims Test Series 2021 )

Answer Justification :

Learning: Coal gasification is the process of producing syngas. Gasification is done using a
mixture of coal, water, air and/or oxygen.

Coal gas or Syngas is a mixture consisting primarily of methane (CH4), carbon monoxide (CO),
hydrogen (H2), carbon dioxide (CO2) and water vapor (H2O)–from coal and water, air and/or
oxygen.

In current practice, large-scale instances of coal gasification are primarily for electricity
generation.

The hydrogen obtained from coal gasification can be used for various purposes such as making
ammonia, powering a hydrogen economy, or upgrading fossil fuels.

Q Source: AR: Page 58: Science 8th NCERT

52. Consider the following statements.


The authors of tawarikh
1. wrote the history of those communities that were often the poorest of all.
2. advised rulers on governance and emphasising the importance of just rule.

Which of the above is/are correct?


A. 1 only
B. 2 only
C. Both 1 and 2
D. None

Correct Answer : B

Answer Justification :

Justification: The authors of Tawarikhs were usually learned men: secretaries,


administrators, poets and courtiers, who both recounted events and advised rulers on
governance, emphasising the importance of just rule.

We must also note that:

the authors of tawarikh lived in cities (mainly Delhi) and hardly ever in villages.
They often wrote their histories for Sultans in the hope of rich rewards.
These authors advised rulers on the need to preserve an “ideal” social order based on
birthright and gender distinctions.
Their ideas were not shared by everybody.

www.insightsactivelearn.com 46
Total Marks : 200
TEST - 3 (Textbook)
( INSTA Prelims Test Series 2021 )

Q Source: Revision: previous test’s syllabus

53. Indianization of the administration, in British India, aimed to


1. Tackle institutionalized racism against Indian candidates
2. Reduce drain of wealth from India
3. Align statues and rules with Indian values

Select the correct answer using the codes below.


A. 2 and 3 only
B. 1 and 2 only
C. 1 only
D. 1, 2 and 3

Correct Answer : B

Answer Justification :

Justification: It has often been said that the Congress in the first twenty years was
“moderate” in its objectives and methods.

During this period it demanded a greater voice for Indians in the government and in
administration.

It demanded that Indians be placed in high positions in the government. For this purpose
it called for civil service examinations to be held in India as well, not just in London.
Since British officers were sending a major part of their large salaries home,
Indianisation, it was hoped, would also reduce the drain of wealth to England.
The demand for Indianization of the administration was part of a movement against
racism, since most important jobs at the time were monopolised by white officials, and
the British generally assumed that Indians could not be given positions of responsibility.
S3 is absurd, there was no such objective.

Q Source: Page 144: History Standard 8th NCERT: Our pasts: Part II

54. Patuas, in nineteenth century India, were famously known for

A. Puppetry
B. Scroll painting
C. Wood carvings
D. Smelting ores

Correct Answer : B

www.insightsactivelearn.com 47
Total Marks : 200
TEST - 3 (Textbook)
( INSTA Prelims Test Series 2021 )

Answer Justification :

Learning: In the nineteenth century a new world of popular art developed in many of the
cities of India.

In Bengal, around the pilgrimage centre of the temple of Kalighat, local village scroll painters
(called patuas) and potters (called kumors in eastern India and kumhars in north India) began
developing a new style of art.

They moved from the surrounding villages into Calcutta in the early 19th Century.

Q Source: Page 132: History Standard 8th NCERT: Our pasts: Part II

55. Consider the following statements about Potti Sreeramulu.


1. He was a follower of Mahatma Gandhi
2. He participated in the individual satyagraha.
3. He was associated with the linguistic reorganization of Andhra region.

Select the correct answer using the codes below.


A. 1 only
B. 1 and 2 only
C. 2 and 3 only
D. 1, 2 and 3

Correct Answer : D

Answer Justification :

Learning: A devout follower of Mahatma Gandhi, he worked much of his life for humanitarian
causes, including support for the Dalit community.

Commenting on Sreeramulu's dedication and fasting ability, Gandhi once said "If only I
have eleven more followers like Sriramulu I will win freedom [from British rule] in a
year.
Between 1941-1942, he participated in the individual satyagraha (before that salt
satyagraha in 1930) and the Quit India movement and was imprisoned on three
occasions.
He was involved in the village reconstruction programmes at Rajkot in Gujarat and
Komaravolu in Krishna district, Andhra Pradesh
In 1952, a popular freedom fighter, Patti Sriramalu, undertook a fast unto death over the
demand for a separate Andhra and expired after fifty-eight days.

www.insightsactivelearn.com 48
Total Marks : 200
TEST - 3 (Textbook)
( INSTA Prelims Test Series 2021 )

His death was followed by three days of rioting, demonstrations, hartals and violence all
over Andhra. The government immediately gave in and conceded the demand for a
separate state of Andhra, which finally came into existence in October 1953.
Simultaneously, Tamil Nadu was created as a Tamil-speaking state.

Q Source: AR: Page 165: History Standard 8th NCERT: Our pasts: Part II

56. Which of these can be used as fire extinguishers?


1. Baking soda
2. Saltpetre
3. Acetylene

Select the correct answer using the codes below.


A. 1 only
B. 1 and 2 only
C. 2 and 3 only
D. 1 and 3 only

Correct Answer : A

Answer Justification :

Justification: S1: Carbon dioxide (CO2) is the best extinguisher. One way to get CO2 is to
release a lot of dry powder of chemicals like sodium bicarbonate (baking soda) or potassium
bicarbonate. Near the fire, these chemicals give off CO2.

S2: Saltpetre is a material used in making explosives, so clearly S2 is incorrect.

S3: Acetylene is a flammable gas and will only increase the fire hazard. So, S3 is wrong.

Q Source: AR: Page 69: Science 8th NCERT

57. Consider the following statements.


1. Tanti from Bengal and the Julahas or Momin weavers from north India are famous for weaving.
2. Aurang was a term used in Medieval India for weavers that worked in workshops.

Which of the above is/are correct?


A. 1 only
B. 2 only
C. Both 1 and 2
D. None

www.insightsactivelearn.com 49
Total Marks : 200
TEST - 3 (Textbook)
( INSTA Prelims Test Series 2021 )

Correct Answer : A

96 pp tre

4
37 n
, 9 tsa en
Answer Justification :

04
86 o
55 ha k C
32 W oo
Learning: S1: Weavers often belonged to communities that specialised in weaving. Their

40 or B
skills were passed on from one generation to the next. The tanti weavers of Bengal, the julahas

26 all ar
or momin weavers of north India, sale and kaikollar and devangs of south India are some of the

88 C um
communities famous for weaving.

K
For coloured textiles, the thread was dyed by the dyer, known as rangrez. For printed cloth
the weavers needed the help of specialist block printers known as chhipigars.

S2: Aurang was a Persian term for a warehouse – a place where goods are collected before
being sold. The term also refers to a workshop.

Q Source: Page 85: History Standard 8th NCERT: Our pasts: Part II

58. Consider the following about International Monetary Fund (IMF) membership.
1. When a country joins the IMF, it is assigned an initial quota in terms of Special Drawing Rights
(SDR) units.
2. A country’s GDP size is irrelevant to the quota it is assigned in IMF.
3. China is the largest member (share-wise) of IMF.

Which of the statement given above is/are correct?


A. 1 and 3 only
B. 3 only
C. 1 only
D. 1 and 2 only

Correct Answer : C

Answer Justification :

Justification: Quotas are denominated in Special Drawing Rights (SDRs), Special Drawing
Rights (SDRs), the IMF’s unit of account. The largest member of the IMF is the United States,

The current quota formula is a weighted average of GDP (weight of 50 percent), openness (30
percent), economic variability (15 percent), and international reserves (5 percent).

For this purpose, GDP is measured through a blend of GDP—based on market exchange rates
(weight of 60 percent)—and on PPP exchange rates (40 percent).

See here https://www.imf.org/external/np/sec/memdir/members.aspx

Q Source: AR: Page 16: Democratic Politics - I

www.insightsactivelearn.com 50
Total Marks : 200
TEST - 3 (Textbook)
( INSTA Prelims Test Series 2021 )

96 pp tre

4
37 n
, 9 tsa en

04
86 o
59. Consider the following about the Veda Samaj, 1864.

55 ha k C
1. It was established in Chennai.

32 W oo
40 or B
2. It promoted widow remarriage and women’s education.

26 all ar
3. Its members believed in the multiplicity of Gods.

88 C um
4. The Samaj condemned superstitions and rituals of orthodox Hinduism.

K
Select the correct answer using the codes below.
A. 1, 3 and 4 only
B. 1, 2 and 4 only
C. 1, 2 and 3 only
D. 2 and 4 only

Correct Answer : B

Answer Justification :

Justification: Veda Samaj was established by Keshab Chandra Sen and K. Sridharalu Naidu
when the former visited Madras in 1864. K. Sridharalu Naidu later visited Calcutta to study
the Brahmo Samaj movement and when he returned, he renamed the Veda Samaj as Brahmo
Samaj of Southern India in 1871.

It was established in Madras (Chennai) in 1864, inspired by the Brahmo Samaj.

It worked to abolish caste distinctions and promote widow remarriage and women’s education.

Its members believed in one God. They condemned the superstitions and rituals of orthodox
Hinduism.

Q Source: Page 120: History Standard 8th NCERT: Our pasts: Part II

60. In the United Nations General Assembly (UNGA), voting shares are distributed between member states on the
basis of

A. Population
B. Contribution to UN
C. Economic Size
D. All members have equal vote

www.insightsactivelearn.com 51
Total Marks : 200
TEST - 3 (Textbook)
( INSTA Prelims Test Series 2021 )

Correct Answer : D

Answer Justification :

Learning: Voting in the General Assembly on important questions, namely, recommendations


on peace and security, budgetary concerns and the election, admission, suspension or
expulsion of members – is by a two-thirds majority of those present and voting.

Other questions are decided by a straightforward majority. Each member country has one
vote.

Apart from approval of budgetary matters, including adoption of a scale of assessment,


Assembly resolutions are not binding on the members.

The Assembly may make recommendations on any matters within the scope of the UN, except
matters of peace and security under Security Council consideration.

The one state, one vote power structure potentially allows states comprising just five percent
of the world population to pass a resolution by a two-thirds vote.

Q Source: Page 15: Democratic Politics - I

61. Consider the following about the National Tiger Conservation Authority (NTCA).
1. It is a statutory body under the Union Ministry of Environment, Forest and Climate change.
2. It approves the Tiger Conservation Plan prepared by the States.
3. It lays down normative standards for tourism activities and guidelines for project tiger from
time to time in national parks.

Which of the above is/are correct?


A. 1 only
B. 2 and 3 only
C. 1 and 3 only
D. 1, 2 and 3

Correct Answer : D

Answer Justification :

Learning: NTCA is a statutory body under the Union Ministry of Environment, Forest and
Climate change. It was provided statutory status by the Wild Life (Protection) Amendment Act,
2006 which had amended Wild Life (Protection) Act, 1972.

Powers and functions of the National Tiger Conservation Authority as prescribed under
section 38O of the Wildlife (Protection) Act, 1972, as amended in 2006 are as under:-

to approve the tiger conservation plan prepared by the State Government under the Act;

www.insightsactivelearn.com 52
Total Marks : 200
TEST - 3 (Textbook)
( INSTA Prelims Test Series 2021 )

evaluate and assess various aspects of sustainable ecology and disallow any ecologically
unsustainable land use such as, mining, industry and other projects within the tiger
reserves;
lay down normative standards for tourism activities and guidelines for project tiger from
time to time for tiger conservation in the buffer and core area of tiger reserves and
ensure their due compliance;

Q Source: Revision: previous test’s syllabus

62. Why was there a controversy surrounding the introduction of the Ilbert Bill, 1883?

A. The bill declared any anti-British organization as illegal and provided for its immediate
closure.
B. The bill increased the revenue assessment of the forthcoming years and imposed a huge
burden on the treasury and people.
C. The bill introduced provisions under which Indian judges could try Europeans in the
court.
D. The bill provided for the confiscation of arms possessed by the Indians.

Correct Answer : C

Answer Justification :

Learning: Ilbert Bill, in the history of India, a controversial measure proposed in 1883 that
sought to allow senior Indian magistrates to preside over cases involving British subjects in
India. The bill, severely weakened by compromise, was enacted by the Indian Legislative
Council on Jan. 25, 1884. The bitter controversy surrounding the measure deepened
antagonism between British and Indians and was a prelude to the formation of the Indian
National Congress the following year.

British subjects in 1873 had been exempted from trial by Indian magistrates, and in cases
involving death or transportation they could only be tried by a high court. But by 1883 the
viceroy, Lord Ripon, proposed to make British subjects amenable to sessions courts, over
which Indians were now senior enough in the civil service to preside.

This proposal as embodied in the Ilbert Bill provoked furious protests, especially among the
Calcutta (Kolkata) European business community and the Bengal indigo planters, and there
was covert sympathy from many officials. A compromise was reached by which a British
subject could claim a jury, half of which would be Europeans. The new Westernized Indian
middle class felt itself slighted by this arrangement, and the incident did much to give Indian
national feeling a political form.

Q Source: Page 142: History Standard 8th NCERT: Our pasts: Part II

www.insightsactivelearn.com 53
Total Marks : 200
TEST - 3 (Textbook)
( INSTA Prelims Test Series 2021 )

63. In its orbital motion, Planet Uranus appears to roll on its side because of

A. a direction of rotation that is opposite to that of the earth


B. highly tilted rotational axis
C. perceived distance from earth
D. a wobbly orbit and poor planetary density

Correct Answer : B

Answer Justification :

Justification: Only B is the correct answer. Uranus rotates from east to west and has a highly
tilted rotational axis. As a result, in its orbital motion it appears to roll on its side. because
the axis of uranus is very tilted, therefore, it appears to roll on it's side. See image below.

Learning: Uranus has an unusual, irregularly shaped magnetosphere. Magnetic fields are
typically in alignment with a planet's rotation, but Uranus' magnetic field is tipped over: the
magnetic axis is tilted nearly 60 degrees from the planet's axis of rotation, and is also offset
from the centre of the planet by one-third of the planet's radius.

Q Source: Page 232: Science 8th NCERT

64. Consider the following statements about a national leader of the independence movement.
1. He led the Salt Satyagraha in south India.
2. He served as a member of the Interim Government of 1946.
3. He was free India’s first Indian Governor-General.

The above refer to?


A. Khan Abdul Ghaffar Khan

www.insightsactivelearn.com 54
Total Marks : 200
TEST - 3 (Textbook)
( INSTA Prelims Test Series 2021 )

B. C. Rajagopalachari
C. Maulana Abdul Kalam Azad
D. E.V. Ramaswamy Naicker

Correct Answer : B

Answer Justification :

Learning: Rajagopalachari was the last Governor-General of India. He also served as leader of
the Indian National Congress, Premier of the Madras Presidency, Governor of West Bengal,
Minister for Home Affairs of the Indian Union and Chief Minister of Madras state.

Rajagopalachari founded the Swatantra Party and was one of the first recipients of India's
highest civilian award, the Bharat Ratna.

He joined the Indian National Congress and participated in the agitations against the Rowlatt
Act, joining the Non-Cooperation movement, the Vaikom Satyagraha, and the Civil
Disobedience movement. In 1930, Rajagopalachari risked imprisonment when he led the
Vedaranyam Salt Satyagraha in response to the Dandi March.

Q Source: Page 156: History Standard 8th NCERT: Our pasts: Part II

65. Consider the following statements about the views of Subhash Chandra Bose.
1. Bose shared the Gandhian ideal of “Ahmisa” and made it a Central pillar of his speeches and
campaigns.
2. Bose considered that industrialization and western domination are factors that will corrode
India and prevent it from becoming strong and self-sufficient.

Which of the above is/are correct?


A. 1 only
B. 2 only
C. Both 1 and 2
D. None

Correct Answer : D

Answer Justification :

Justification: Statement 1: Bose stated, "I am convinced that if we do desire freedom we


must be prepared to wade through blood", a statement at odds with Gandhi's philosophy of
non-violence.

He thus raised the Azad Hind Fauj or the Indian National Army (INA), to free India from
British control.

www.insightsactivelearn.com 55
Total Marks : 200
TEST - 3 (Textbook)
( INSTA Prelims Test Series 2021 )

Subhas Chandra Bose's political views were in support of complete freedom for India at the
earliest, whereas most of the Congress Committee wanted it in phases, through a Dominion
status

Statement 2: Gandhi was hostile to industrialisation, while Bose advocated industrialization,


perhaps inspired by soviet progress.

Even though Bose and Mohandas K. Gandhi had differing ideologies, the latter called Bose the
"Prince among the Patriots" in 1942. Bose admired Gandhi, recognising his importance as a
symbol of Indian nationalism; he called him "The Father of Our Nation" in a radio broadcast.

Q Source: AR: Page 155: History Standard 8th NCERT: Our pasts: Part II

66. Consider the following about the Sufis.


1. They rejected outward religiosity and emphasised love and devotion to God.
2. It rejected idol worship.
3. The Sufis only adopted the rituals and codes of behaviour as prescribed by the Muslim ulamas
and religious scholars.

Select the correct answer using the codes below.


A. 1 and 2 only
B. 2 and 3 only
C. 1 and 3 only
D. 1, 2 and 3

Correct Answer : A

Answer Justification :

Justification: Statement 1: Sufis were Muslim mystics. They rejected outward religiosity and
emphasised on compassion towards all fellow human beings.

Statement 2: Islam propagated strict monotheism or submission to one God. It also rejected
idol worship and considerably simplified rituals of worship into collective prayers.

Statement 3: At the same time Muslim scholars developed a holy law called Shariat. The Sufis
often rejected the elaborate rituals and codes of behaviour demanded by Muslim religious
scholars.They sought union with God much as a lover seeks his beloved with a disregard for
the world.

Like the Nathpanthis, Siddhas and Yogis, the Sufis too believed that the heart can be trained
to look at the world in a different way.

They developed elaborate methods of training using zikr (chanting of a name or sacred
formula), contemplation, sama (singing), raqs (dancing), discussion of parables, breath control,
etc. under the guidance of a master or pir.
www.insightsactivelearn.com 56
Total Marks : 200
TEST - 3 (Textbook)
( INSTA Prelims Test Series 2021 )

Thus emerged the silsilas, a genealogy of Sufi teachers, each following a slightly different
method (tariqa) of instruction and ritual practice.

Q Source: Revision: previous test’s syllabus

67. D.K. Basu Guidelines laid down by the Supreme Court are primarily applicable to

A. bonder labour resettlement


B. sexual harassment at workplace
C. rehabilitation of manual scavengers
D. arrest and detention by police

Correct Answer : D

Answer Justification :

Justification: The Supreme Court of India has laid down specific requirements and
procedures that the police and other agencies have to follow for the arrest, detention and
interrogation of any person.

These are known as the D.K. Basu Guidelines and some of these include:

The police officials who carry out the arrest or interrogation should wear clear, accurate
and visible identification and name tags with their designations;
A memo of arrest should be prepared at the time of arrest and should include the time
and date of arrest.
It should also be attested by at least one witness who could include a family member of
the person arrested. The arrest memo should be counter-signed by the person arrested.
When a friend or relative lives outside the district, the time, place of arrest and venue of
custody must be notified by police within 8 to 12 hours after arrest.
The person arrested must be made aware of his right to have someone informed of his
arrest or detention as soon as he is put under arrest or is detained.

Q Source: Revision: previous test’s syllabus

68. Consider the following statements about Ryotwari system.


1. A pilot was implemented by Captain Alexander Read in some areas in Eastern India first.
2. The system was developed and extended by Thomas Munro all over India.

Which of the above is/are correct?


A. 1 only

www.insightsactivelearn.com 57
Total Marks : 200
TEST - 3 (Textbook)
( INSTA Prelims Test Series 2021 )

B. 2 only
C. Both 1 and 2
D. None

Correct Answer : D

Answer Justification :

Justification: Ryotwari was tried first on a small scale by Captain Alexander Read in some of
the areas that were taken over by the Company after the wars with Tipu Sultan.

Subsequently developed by Thomas Munro, this system was gradually extended all over
south India.
Read and Munro felt that in the south there were no traditional zamindars. The
settlement, they argued, had to be made directly with the cultivators (ryots) who had
tilled the land for generations.
Their fields had to be carefully and separately surveyed before the revenue assessment
was made. Munro thought that the British should act as paternal father figures
protecting the ryots under their charge

Q Source: Revision: previous test’s syllabus

69. The Indian National Congress demanded a Constituent Assembly for the first time during

A. The first Round Table conference


B. “Purna awaraj declaration” at Lahore session
C. Quit India movement
D. None of the above

Correct Answer : D

Answer Justification :

Learning: In June 1934, for the first time, the Indian National Congress formally demanded
that a Constituent Assembly composed of Indians be set up to frame a Constitution for India.
With this, the Congress added a new strand to its political engagement with the British on the
future of India. Going forward, the demand for Constituent would be made alongside calls for
‘Purna Swaraj’ or complete freedom.

The trigger for the Congress’s demand was the 1934 British White Paper. The Round Table
Conferences organised to resolve India’s constitutional and communal issues, had failed.
Following this, the British government, without consulting Indian political leaders, drafted a

www.insightsactivelearn.com 58
Total Marks : 200
TEST - 3 (Textbook)
( INSTA Prelims Test Series 2021 )

set of constitutional reforms. These were widely criticised and attacked by Indian political
parties. The Congress now felt that the only way forward was for Indians themselves to decide
their constitutional future.

On June 16-17, 1934 the Congress Working Committee passed a resolution which stated that
the White Paper ‘in no way expresses the will of the people of India…the only satisfactory
alternative…is constitution drawn up by a Constituent Assembly elected on the basis of adult
suffrage or as near it as possible, with the power, if necessary, to the important minorities to
have their representative elected exclusively by the electors belonging to such minorities’

Then in 1935, the British passed the Government of India Act, which was the culmination of
the process that began with the White Paper. The Government of India Act 1935 was in effect
going to be the Constitution of India. The Congress rejected the Act and in a 1936 resolution
declared that ‘no constitution imposed by outside authority and no constitution which curtails
the sovereignty of the people of india and which does not recognise their right to shape and
control fully their political and economic future can be accepted. ‘

Q Source: Revision: previous test’s syllabus

70. In Organic Farming


1. No genetic modification or GM crops are used to increase the farm yield.
2. No manures, whether natural or chemical, are used to modify the natural properties of the
soil.
3. No kind of pesticides or weedicides are used in the farm.

Select the correct answer using the codes below.


A. 1 and 3 only
B. 2 and 3 only
C. 1 only
D. 1, 2 and 3

Correct Answer : C

Answer Justification :

Justification: Organic farming is a system which avoids or largely excludes the use of
synthetic inputs (such as fertilizers, pesticides, hormones, feed additives etc) and to the
maximum extent feasible rely upon crop rotations, crop residues, animal manures, off-farm
organic waste, mineral grade rock additives and biological system of nutrient mobilization and
plant protection.

Statement 1: It avoids the use of GM seeds to reduce complications on the field and uses
naturally obtained seeds.

Statement 2: It does not use chemical fertilizers, but uses organic manures to boost the

www.insightsactivelearn.com 59
Total Marks : 200
TEST - 3 (Textbook)
( INSTA Prelims Test Series 2021 )

productivity and nutrient base of the soil. So, 2 is incorrect.

Statement 3: Similarly, it uses natural methods of pest control, and natural weedicides and
herbicides, and not chemical ones. So, statement 3 is incorrect.

Q Source: Revision: previous test’s syllabus

71. Coffee is best grown in which of these conditions?

A. Heavy rainfall conditions with alluvial soil


B. Dry climate with sandy soil
C. Warm, wet climate and well-drained loamy soil
D. Cold and frost with minimal sunshine

Correct Answer : C

Answer Justification :

Learning: Coffee requires warm and wet climate and well-drained loamy soil. Hill slopes are
more suitable for growth of this crop.

Ideal climatic conditions to grow coffee are related to temperature and rainfall; temperatures
in the range of 73 °F (23 °C) and 82 °F (28 °C) with rainfall incidence in the range of 60–80
inches (1.5–2.0 m) followed by a dry spell of 2–3 months suit the Arabica variety.

Brazil is the leading producer followed by Columbia and India.

Cold temperatures closer to freezing conditions are not suitable to grow coffee. Regions where
the rainfall is less than 40 inches (1.0 m), providing irrigation facilities is essential. In the
tropical region of the south Indian hills, these conditions prevail leading to coffee plantations
flourishing in large numbers.

Q Source: Revision: previous test’s syllabus

72. Which of the following was the motivating reason behind the Khilafat agitation in British India?

A. Partition of Bengal
B. A harsh treaty being imposed on the Turkish Sultan
C. Jallianwala massacre
D. Enactment of the Indian Defence Act

www.insightsactivelearn.com 60
Total Marks : 200
TEST - 3 (Textbook)
( INSTA Prelims Test Series 2021 )

Correct Answer : B

Answer Justification :

Learning: In 1920 the British imposed a harsh treaty on the Turkish Sultan or Khalifa. People
were furious about this as they had been about the Jallianwala massacre.

Also, Indian Muslims were keen that the Khalifa be allowed to retain control over Muslim
sacred places in the erstwhile Ottoman Empire.
The leaders of the Khilafat agitation, Mohammad Ali and Shaukat Ali, now wished to
initiate a full-fledged Non-Cooperation Movement.
Gandhiji supported their call and urged the Congress to campaign against “Punjab
wrongs” (Jallianwala massacre), the Khilafat wrong and demand swaraj

Q Source: Page 149: History Standard 8th NCERT: Our pasts: Part II

73. Poles will not experience about six months day and six months night if earth did NOT have this feature:

A. Axial Tilt
B. Bulge at the poles
C. A natural satellite
D. Bulge at the equator

Correct Answer : A

Answer Justification :

Justification: Earth is tilted in such a way that even after one complete rotation North Pole
will remain dark as it is tilted away from the Sun. See image below.

If the earth did not revolve around the Sun, but only rotated on its axis, we would see one Pole
of the earth permanently facing day or permanently facing night. It would never change. You
can visualize this.

So, revolution of the earth around the Sun changes the direction of the tilt that faces the Sun,
and hence brings day/night to the other pole.

www.insightsactivelearn.com 61
Total Marks : 200
TEST - 3 (Textbook)
( INSTA Prelims Test Series 2021 )

Q Source: Revision: previous test’s syllabus

74. The Swadeshi movement encouraged the idea(s) of


1. Self-help and self-dependence
2. National education
3. Use of Indian languages

Which of the above is/are correct?


A. 1 only
B. 2 and 3 only
C. 1 and 3 only
D. 1, 2 and 3

Correct Answer : D

Answer Justification :

Learning: It sought to oppose British rule and encourage the ideas of self-help, Swadeshi
enterprise, national education, and use of Indian languages.

To fight for swaraj, the radicals advocated mass mobilisation and boycott of British institutions
and goods.

Some individuals also began to suggest that “revolutionary violence” would be necessary to
overthrow British rule.

However, Swadeshi movement was not clear about the economic model based on which these
objectives are to be achieved. Socialist model in India was talked about much later when
Nehru engaged in the INM actively and after the Russian revolution.

www.insightsactivelearn.com 62
Total Marks : 200
TEST - 3 (Textbook)
( INSTA Prelims Test Series 2021 )

We will cover the movement in detail later.

Q Source: AR: Page 146: History Standard 8th NCERT: Our pasts: Part II

75. Consider the following about Lord Macaulay and his contributions.
1. He emphasised the need to teach vernacular languages in British India.
2. He issued orders for the promotion of Oriental institutions like the Calcutta Madrasa and
Benaras Sanskrit College.
3. The English Education Act of 1835 was introduced close to the presentation of ‘Macaulay’s
minute’

Select the correct answer using the codes below.


A. 1 and 3 only
B. 3 only
C. 2 only
D. 2 and 3 only

Correct Answer : B

Answer Justification :

Justification: Statement 1: Macaulay emphasised the need to teach the English language.

He felt that knowledge of English would allow Indians to read some of the finest literature the
world had produced; it would make them aware of the developments in Western science and
philosophy.

Teaching of English could thus be a way of civilising people, changing their tastes, values and
culture.

Statement 2: A decision was taken to make English the medium of instruction for higher
education, and to stop the promotion of Oriental institutions like the Calcutta Madrasa and
Benaras Sanskrit College.

Statement 3: Macaulay presented his ‘Minute on Indian Education’ that sought to establish
the need to impart English education to Indian ‘natives’.

The English Education Act 1835 was a legislative Act of the Council of India, gave effect to a
decision in 1835 by Lord William Bentinck, then Governor-General of the British East India
Company, to reallocate funds it was required by the British Parliament to spend on education
and literature in India.

In discussions leading up to the Act Thomas Babington Macaulay produced his famous
Memorandum on (Indian) Education which was scathing on the inferiority (as he saw it) of
native (particularly Hindu) culture and learning. He argued that Western learning was
superior, and currently could only be taught through the medium of English. There was
www.insightsactivelearn.com 63
Total Marks : 200
TEST - 3 (Textbook)
( INSTA Prelims Test Series 2021 )

therefore a need to produce—by English-language higher education—"a class of persons,


Indian in blood and colour, but English in taste, in opinions, in morals and in intellect" who
could in their turn develop the tools to transmit Western learning in the vernacular languages
of India.

Q Source: Page 98: History Standard 8th NCERT: Our pasts: Part II

76. High tides in the Ocean are helpful for


1. navigation as they allow the ships to come close to the harbour
2. fishing since they bring more fish closer to the shore
3. supporting inter-tidal aquatic communities

Select the correct answer using the codes below.


A. 1 and 2 only
B. 2 and 3 only
C. 1 and 3 only
D. 1, 2 and 3

Correct Answer : D

Answer Justification :

Justification: Statement 1: High tides raise the water level close to the shores. This helps the
ships to arrive at the harbour more easily.

Statement 2: The high tides also help in fishing. Many more fish come closer to the shore
during the high tide. This enables fishermen to get a plentiful catch.

Statement 3: The intertidal zone (sometimes referred to as the littoral zone) is the area that is
exposed to the air at low tide and underwater at high tide (the area between the low and
high tide lines). This area can include many different types of habitats, including steep rocky
cliffs, sandy beaches, or wetlands.

Organisms in the intertidal zone are adapted to an environment of harsh extremes. The
intertidal zone is also home to several species from different phyla (Porifera, Annelida,
Coelenterata, Mollusca, Arthropoda, etc.). Water is available regularly with the tides, but
varies from fresh with rain to highly saline and dry salt, with drying between tidal inundations.
Wave splash can dislodge residents from the littoral zone.

Q Source: Revision: previous test’s syllabus

77. In Ancient India, Shrenis of merchants organized trade. Consider the following about them.
1. Shrenis served as banks where only men were allowed to make deposits.

www.insightsactivelearn.com 64
Total Marks : 200
TEST - 3 (Textbook)
( INSTA Prelims Test Series 2021 )

2. Shrenis vocally opposed religious institutions and ritual fanaticism.

Select the correct answer using the codes below.


A. 1 only
B. 2 only
C. Both 1 and 2
D. None

Correct Answer : D

Answer Justification :

Justification: Many crafts persons and merchants formed associations known as shrenis.
These shrenis of crafts persons provided training, procured raw material, and distributed the
finished product.

Then shrenis of merchants organised the trade. Shrenis also served as banks, where rich men
and women deposited money.

This was invested, and part of the interest was returned or used to support religious
institutions such as monasteries.

Q Source: Revision: previous test’s syllabus

78. Consider the following about Raja Ram Mohan Roy.


1. He was keen to spread the knowledge of Western education in the country.
2. He showed that the practice of sati had no sanction in ancient texts.
3. He was given the title of “Raja” by Lord Irwin, a Viceroy of British India.

Select the correct answer using the codes below.


A. 1 and 2 only
B. 2 and 3 only
C. 1 and 3 only
D. 2 only

Correct Answer : A

Answer Justification :

Justification: Statement 1: S1 has been quoted directly from NCERT, so even if it seems a bit
subjective, it is alright! With the spread of western education he wanted to bring about greater
freedom and equality for women.

Statement 2: He tried to show through his writings that the practice of widow burning had no

www.insightsactivelearn.com 65
Total Marks : 200
TEST - 3 (Textbook)
( INSTA Prelims Test Series 2021 )

sanction in ancient text.

The strategy adopted by Rammohun was used by later reformers as well. Whenever they
wished to challenge a practice that seemed harmful, they tried to find a verse or sentence in
the ancient sacred texts that supported their point of view. They then suggested that the
practice as it existed at present was against early tradition.

Statement 3: He was given the title of Raja by Akbar II, the Mughal emperor. His influence
was apparent in the fields of politics, public administration, education and religion. Raja Ram
Mohan Roy is considered to be the "Father of the Indian Renaissance" by many historians.

Q Source: Page 110: History Standard 8th NCERT: Our pasts: Part II

79. The United Nations Secretary General is appointed by the UN General Assembly only after the
recommendation of the

A. Former UN Secretary General


B. UN Security Council
C. UN Social and Economic Council
D. International Court of Justice

Correct Answer : B

Answer Justification :

Justification & Learning: The Secretary-General is appointed by the General Assembly only
based on the recommendation of the UNSC.

This implies that essentially the P5 chooses the Sec. Gen.

There is an informal understanding between the members that Sec. Gen. should alternatively
be from Western and Eastern Bloc.

Although there is technically no limit to number of five-year terms a Secretary-General may


serve, none so far has held office for more than two terms.

Q Source: AR: Page 15: Democratic Politics – I

80. Consider the following statements.


1. The Prarthana Samaj adhered to the Bhakti tradition.
2. Paramhans Mandali worked for the abolition of caste and caste-based inequalities.
3. Christian Missionaries of the 19th Century did not recognize the caste based traditions of

www.insightsactivelearn.com 66
Total Marks : 200
TEST - 3 (Textbook)
( INSTA Prelims Test Series 2021 )

India and considered them irrelevant.

Select the correct answer using the codes below.


A. 1 and 2 only
B. 2 and 3 only
C. 1 only
D. 1, 2 and 3

Correct Answer : A

Answer Justification :

Justification: Statement 1: Prarthana Samaj was founded by Atmaram Panduranga (with the
help of Keshav Chandra Sen) in 1867 with an aim to make people believe in one God and
worship only one God. It became popular after Mahadev Govind Ranade joined. The main
reformers were the intellectuals who advocate reforms of the social system of the Hindus.

Statement 2: It was founded in Bombay in 1840s by Dadoba Panderung (1842-82) and Bal
Shastri Jambhekar of Maharashtra in 1849.

It said God alone should be worshipped and real religion is based on love and moral conduct.

Statement 3: During the course of the nineteenth century, Christian missionaries began
setting up schools for tribal groups and “lower”-caste children. One of the most vocal amongst
the “low-caste” leaders was Jyotirao Phule. He studied in schools set up by Christian
missionaries. So, S3 is wrong.

Q Source: Page 114: History Standard 8th NCERT: Our pasts: Part II

81. Consider the following statements


1. A natural harbour is a landform where a part of a body of water is protected and deep enough
to furnish anchorage.
2. Irregular and indented coastlines, like that on the Atlantic Ocean, provide an ideal condition
for natural harbours.

Which of the above is/are correct?


A. 1 only
B. 2 only
C. Both 1 and 2
D. None

Correct Answer : C

Answer Justification :

www.insightsactivelearn.com 67
Total Marks : 200
TEST - 3 (Textbook)
( INSTA Prelims Test Series 2021 )

Justification: A natural harbour is a landform where a part of a body of water is protected


and deep enough to furnish anchorage.

Irregular and indented coastlines provide the perfect condition for natural harbours. Atlantic
ocean hosts some of the most indented and the best harbours in the world, and consequently is
also one of the busiest in the World.

Natural harbours have long been of great strategic naval and economic importance, and many
great cities of the world are located on them.

Having a protected harbour reduces or eliminates the need for breakwaters as it will result in
calmer waves inside the harbour.

Q Source: Revision: previous test’s syllabus

82. Match these parties, with the people who were associated with it in various capacities:
1. Swantantra party: Kanhaiyalal Maniklal Munshi
2. Republican Party of India: B.R. Ambedkar
3. Hindu Mahasabha: Shyama Prasad Mukherjee

Select the correct answer using the codes below.


A. 1 and 3 only
B. 2 only
C. 1 and 2 only
D. 1, 2 and 3

Correct Answer : D

Answer Justification :

Justification: Statement 1: Kanaiyalal Munshi was NOT one of the founders of Swantantra
party – he joined in later. C. Rajagopalachari was the founder.

Statement 2: Ambedkar founded the Republican Party of India. He also served as the
Chairman of the Drafting Committee of the Constitution.

Statement 3: Shyama Prasad Mukherjee who was Active in Hindu Mahasabha and later
became the founder President of Bharatiya Jansangh (now BJP).

More on these later as we cover more standard history textbooks.

Q Source: AR: Page 48: Democratic Politics - I

83. Consider the following statements.

www.insightsactivelearn.com 68
Total Marks : 200
TEST - 3 (Textbook)
( INSTA Prelims Test Series 2021 )

1. Gangetic river dolphin is India’s National Aquatic species.


2. Project Dolphin will cover both freshwater (river) and marine (oceanic) dolphins.
3. Ganges river dolphins are protected under Schedule I of Wildlife Protection Act (1972).

Select the correct answer using the codes below.


A. 1 only
B. 1, 2 and 3
C. 2 and 3 only
D. 1 and 3 only

Correct Answer : B

Answer Justification :

Justification: Project Dolphin will cover both freshwater (river) and marine (oceanic)
dolphins.

Gangetic River Dolphin

Gangetic river dolphin, declared as a National Aquatic species in 2010, is a species of


freshwater dolphins primarily found in the Ganges and Brahmaputra rivers and their
tributaries in India, Bangladesh and Nepal.
At present, there are about 3,700 Gangetic river dolphins in the Indian River systems.
These dolphins are sighted along deep river reaches in Assam, Bihar, Jharkhand,
Madhya Pradesh, Rajasthan, Uttar Pradesh and West Bengal.
River Dolphin has two sub-species namely Indus River Dolphins and Gangetic River
Dolphins, both subspecies are listed by the IUCN as endangered.
Ganges river dolphins are protected under Schedule I of Wildlife Protection Act (1972).

Q Source: Current events

84. Consider the following statements about PM Cares Fund and PM National Relief Fund.
1. While the former was setup to handle pandemics like COVID-19, the latter deals with all kinds
of natural disasters.
2. While the former is entirely donation based, the latter receives regular statutory budget
support.

Select the correct answer using the codes below.


A. 1 only
B. 2 only
C. Both 1 and 2

www.insightsactivelearn.com 69
Total Marks : 200
TEST - 3 (Textbook)
( INSTA Prelims Test Series 2021 )

D. None

Correct Answer : A

Answer Justification :

Justification: PM-Citizen Assistance and Relief in Emergency Situations (PM – CARES) Fund
is set up to support the government in its fight against the impact of the coronavirus
pandemic.

The fund consists entirely of voluntary contributions from individuals/organizations and


does not get any budgetary support.

Key Differences/similarities between Prime Minister's National Relief Fund and PM-CARES are
as follows:

Both PM- National Relief Fund and PM-CARES fund disaster support. For instance, PM
CARES aims “To undertake and support relief or assistance of any kind relating to a
public health emergency or any other kind of emergency, calamity or distress, either
man-made or natural, including the creation or upgradation of healthcare or
pharmaceutical facilities, other necessary infrastructure, funding relevant research or
any other type of support”.
In pursuance of an appeal by the then Prime Minister, Pt. Jawaharlal Nehru in January,
1948, the Prime Minister’s National Relief Fund (PMNRF) was established with public
contributions to assist displaced persons from Pakistan. The resources of the PMNRF are
now utilized primarily to render immediate relief to families of those killed in natural
calamities like floods, cyclones and earthquakes, etc. and to the victims of the major
accidents and riots. Assistance from PMNRF is also rendered, to partially defray the
expenses for medical treatment like heart surgeries, kidney transplantation, cancer
treatment and acid attack etc. The fund consists entirely of public contributions and does
not get any budgetary support. The corpus of the fund is invested in various forms with
scheduled commercial banks and other agencies. Disbursements are made with the
approval of the Prime Minister. PMNRF has not been constituted by the Parliament.
PM CARES is a donation-based fund and the legislative hurdles for withdrawal from the
funds are absent. Prime Minister's National Relief Fund (PMNRF) was also established
entirely with public contributions and does not get any budgetary support. PMNRF
accepts voluntary contributions from Individuals, Organizations, Trusts, Companies and
Institutions etc. All contributions towards PMNRF are exempt from Income Tax under
section 80(G).
The PM CARES Fund tries to differentiate itself from PMNRF by enabling micro-
donations. One can donate as low as Rs 10 in the PM CARES Fund. Minimum donation
received under PM National Relief Fund is Rs 100.

Q Source: In news frequently

www.insightsactivelearn.com 70
Total Marks : 200
TEST - 3 (Textbook)
( INSTA Prelims Test Series 2021 )

85. If the majority of the Lok Sabha members approve that they have ‘no confidence’ in the Council of Ministers
1. All ministers including the Prime Minister must resign
2. Lok Sabha is immediately dissolved.

Which of the above is/are correct?


A. 1 only
B. 2 only
C. Both 1 and 2
D. None of the above

Correct Answer : A

Answer Justification :

Justification: A no-confidence motion leads to the resignation of the entire Council of


Ministers (including the PM as she leads the council of Ministers).

But, it does not lead to the immediate dissolution of the Lok Sabha. The Lok Sabha is not
dissolved by the President if an alternative government can be formed.

If the government cannot be formed at all, then the President dissolves the Lok Sabha. So, S2
is wrong.

Q Source: Page 85: Democratic Politics - I

86. Participatory Notes or P-Notes (PNs) are of concern in the Indian financial sector due to

A. Cyber fraud
B. Bank defaults
C. Money laundering
D. Poor fund flow

Correct Answer : C

Answer Justification :

Justification: Participatory Notes or P-Notes (PNs) are financial instruments issued by a


registered foreign institutional investor (FII) to an overseas investor who wishes to invest in
Indian stock markets without registering themselves with the market regulator, the Securities
and Exchange Board of India (SEBI).

1. P-Notes are Offshore Derivative Investments (ODIs) with equity shares or debt securities
as underlying assets.

www.insightsactivelearn.com 71
Total Marks : 200
TEST - 3 (Textbook)
( INSTA Prelims Test Series 2021 )

2. They provide liquidity to the investors as they can transfer the ownership by
endorsement and delivery.
3. While the FIIs have to report all such investments each quarter to SEBI, they need not
disclose the identity of the actual investors.

What are govt & regulator’s concerns?

The primary reason why P-Notes are worrying is because of the anonymous nature of the
instrument as these investors could be beyond the reach of Indian regulators.

Further, there is a view that it is being used in money laundering among wealthy Indians, like
the promoters of companies, using it to bring back unaccounted funds and to manipulate their
stock prices.

Q Source:
https://www.insightsonindia.com/2020/08/22/insights-daily-current-affairs-pib-summary-22-aug
ust-2020/

87. Consider the following about polyphenols.


1. They impart vivid colours to fruits and vegetables
2. They are used in Ayurvedic medicine
3. They are known as powerful anti-oxidants

Select the correct answer using the codes below


A. 2 only
B. 1 and 3 only
C. 1, 2 and 3
D. 2 and 3 only

Correct Answer : C

Answer Justification :

Justification: Flavonoids are part of the polyphenol class of phytonutrients (plant chemicals)
found in almost all fruits and vegetables.

Along with carotenoids (organic pigments), they are responsible for the vivid colours in
fruits and vegetables.
Polyphenols have historically been used in Chinese and Ayurvedic medicine and are
associated with skin protection, brain function, blood sugar and blood pressure
regulation.
These are powerful antioxidants with anti-inflammatory and immune system benefits as
well.
A diet rich in flavonoids can protect from diseases related to heart, liver, kidney, brain
and other infectious diseases.

www.insightsactivelearn.com 72
Total Marks : 200
TEST - 3 (Textbook)
( INSTA Prelims Test Series 2021 )

Recently, scientists from Agharkar Research Institute (ARI), found the first synthetic
route for producing flavonoids molecules related to the treatment of tuberculosis and
chikungunya.
It will ensure their availability at all seasons without the dependence on natural timings
of the plantations.
It is for the first time that scientists have been able to synthesize the flavonoid molecules
such as rugosa flavonoids, podocare flavone and isoflavone in a lab.
These three molecules have so far been isolated from plants only and are found to inhibit
tuberculosis and chikungunya.

Q Source: Current events

88. Millennium Alliance, recently seen in news, is an initiative/organization concerned with

A. Cross-border terrorism
B. Implementation of Sustainable Development Goals
C. Identification, testing and scaling of Indian innovations
D. Financing of NBFCs

Correct Answer : C

Answer Justification :

Justification: Millennium Alliance Round 6 & COVID19 Innovation Challenge-Award


Ceremony was recently held. 49 innovations in 5 focus areas were recognised.

What is Millennium Alliance?

It is an innovation-driven and impact-focused initiative leveraging collaborative resources to


identify test and scale Indian innovations that address global development solutions.

It is a consortium of partners (Public-Private Partnership) including the Department of Science


and Technology, Govt. of India, United States Agency for International Development (USAID),
Federation of Indian Chambers of Commerce and Industry (FICCI), UK Government’s
Department for International Development (DFID), Facebook and Marico Innovation
Foundation.

Q Source:
https://www.insightsonindia.com/2020/08/20/insights-daily-current-affairs-pib-summary-20-aug
ust-2020/

www.insightsactivelearn.com 73
Total Marks : 200
TEST - 3 (Textbook)
( INSTA Prelims Test Series 2021 )

89. Positive Pay Mechanism introduced by the RBI, seen in news recently, is a financial mechanism to

A. reduce overhead banking costs


B. peer-to-peer lending
C. help stressed MSMEs
D. detect cheque frauds

Correct Answer : D

Answer Justification :

Justification: Positive Pay is a fraud detection tool adopted by banks to protect customers
against forged, altered or counterfeit cheques.

It cross verifies all details of the cheque issued before funds are encashed by the
beneficiary.
In case of a mismatch, the cheque is sent back to the issuer for examination.
By following such a system, a bank knows of a cheque being drawn by the customer even
before it is deposited by the beneficiary into his/her account.
RBI has introduced a new 'Positive Pay' mechanism which will make cheque payments
safer and reduce instances of frauds.
Under the new system Issuers will be able to send all details to their bank, thereby
ensuring faster clearance of cheques above Rs 50,000.
All cheques will be processed as per the information sent by the account holder at the
time of issuance of cheques.
This will cover approximately 20 per cent of transactions by volume and 80 per cent by
value.

Q Source: Current events

90. Which of these terms find mention in the Preamble of the Constitution?

A. Liberalism
B. Socialism
C. Globalism
D. Capitalism

Correct Answer : B

Answer Justification :

www.insightsactivelearn.com 74
Total Marks : 200
TEST - 3 (Textbook)
( INSTA Prelims Test Series 2021 )

Justification: The Indian brand of socialism is a ‘democratic socialism’ and not a


‘communistic socialism’ (also known as ‘state socialism’) which involves the nationalisation of
all means of production and distribution and the abolition of private property.

Democratic socialism, on the other hand, holds faith in a ‘mixed economy’ where both public
and private sectors co-exist side by side.

As the Supreme Court says, ‘Democratic socialism aims to end poverty, ignorance, disease and
inequality of opportunity. Indian socialism is a blend of Marxism and Gandhism, leaning
heavily towards Gandhian socialism’.

The new economic policy (1991) of liberalisation, privatisation and globalisation has, however,
diluted the socialist credentials of the Indian State. These terms however don’t find mention in
the constitution.

Q Source: Chapter 3: Democratic Politics - I

91. Atal Ranking of Institutions on Innovation Achievements (ARIIA) is an initiative of Ministry of Human
Resource Development (MHRD), Govt. of India. Consider the following about it.
1. ARIIA systematically ranks all major higher educational institutions and universities in India.
2. ARIA calculation is based on indicators related to “Innovation and Entrepreneurship
Development” amongst students and faculties.
3. It measures the impact created by these innovations and Startups from Higher Educational
Institutions (HEIs) in society and market.

Select the correct answer using the codes below.


A. 1 only
B. 2 and 3 only
C. 2 only
D. 1, 2 and 3

Correct Answer : D

Answer Justification :

Justification: ARIIA systematically ranks all major higher educational institutions and
universities in India.

The calculation is based on indicators related to “Innovation and Entrepreneurship


Development” amongst students and faculties.
It also measure the impact created by these innovations and Startups from Higher
Educational Institutions (HEIs) in society and market.
It aims to uplifting India’s Position in the Global Innovation Index from 52th to top 30 in
a time period of 5 Years.
ARIIA metrics considers all major parameters used globally to rank education

www.insightsactivelearn.com 75
Total Marks : 200
TEST - 3 (Textbook)
( INSTA Prelims Test Series 2021 )

institutions and some parameters which are India specific in nature.


The parameters are primarily organized in major broad parameters with 7 indicators.
Scores is computed based on the weights allotted to each head, overall score can take a
maximum value of 100.
All recognized Higher Educational Institutions of India are eligible to participate in the
ARIIA ranking.
In the recent ARIIA IIT Madras has bagged the top position under the category of
Institutes of National Importance, Central Universities.

Q Source: Current events

92. Consider the following statements.


During the election period
1. The Election Commission of India (ECI) can order the government to observe election related
guidelines.
2. ECI implements the Code of Conduct and can punish any candidate or party that violates it.

Which of the above is/are correct?


A. 1 only
B. 2 only
C. Both 1 and 2
D. None

Correct Answer : C

Answer Justification :

Justification: Very few election commissions in the world have such wide-ranging powers as
the Election Commission of India.

EC takes decisions on every aspect of conduct and control of elections from the
announcement of elections to the declaration of results.
It implements the Code of Conduct and punishes any candidate or party that violates it.
During the election period, the EC can order the government to follow some guidelines,
to prevent use and misuse of governmental power to enhance its chances to win
elections, or to transfer some government officials.
We will cover more on it in coming tests.

Q Source: Page 69: Democratic Politics - I

www.insightsactivelearn.com 76
Total Marks : 200
TEST - 3 (Textbook)
( INSTA Prelims Test Series 2021 )

93. Consider the following statements about Prime Minister Employment Generation Program (PMEGP).
1. PMEGP is a central sector scheme administered by the Ministry of Micro, Small and Medium
Enterprises (MoMSME).
2. Small Industries Development Bank of India (SIDBI) is the national level nodal agency.
3. It is a credit-linked subsidy scheme which promotes self-employment through setting up of
micro-enterprises

Select the correct answer using the codes below.


A. 1 only
B. 1 and 3 only
C. 2 and 3 only
D. 1, 2 and 3

Correct Answer : B

Answer Justification :

Justification: PMEGP is a central sector scheme administered by the Ministry of Micro, Small
and Medium Enterprises (MoMSME).

Launched in 2008-09, it is a credit-linked subsidy scheme which promotes self-employment


through setting up of micro-enterprises, where subsidy up to 35% is provided by the
Government through Ministry of MSME for loans up to ₹25 lakhs in manufacturing and ₹10
lakhs in the service sector.

Implementation:

National Level- Khadi and Village Industries Commission (KVIC) as the nodal agency.

State Level- State KVIC Directorates, State Khadi and Village Industries Boards (KVIBs),
District Industries Centres (DICs) and banks.

Eligibility:

1. Any individual above 18 years of age, Self Help Groups, Institutions registered
under Societies Registration Act 1860, Production Co-operative Societies and Charitable
Trusts are eligible.
2. Existing Units and the units that have already availed Government Subsidy under any
other scheme of Government of India or State Government are not eligible.
3. Only new projects are considered for sanction under PMEGP.

Q Source:
https://www.insightsonindia.com/2020/08/22/insights-daily-current-affairs-pib-summary-22-aug
ust-2020/

www.insightsactivelearn.com 77
Total Marks : 200
TEST - 3 (Textbook)
( INSTA Prelims Test Series 2021 )

94. Consider the following about Science and Technology Indicators (STI) compendium.
1. It is a periodic compendium of the state of scientific research in India.
2. It is prepared by the Department of Science Technology.

Select the correct answer using the codes below.


A. 1 only
B. 2 only
C. Both 1 and 2
D. None

Correct Answer : C

Answer Justification :

Justification: Science and Technology Indicators (STI) is a periodic compendium of the state
of scientific research in India.

It is prepared by the National Science and Technology Management Information System,


a division of the Department of Science Technology.
According to highlights of Science and Technology Indicators (STI), 2018,
1. India’s private sector research companies appear to employ a larger proportion of
women in core research and development activities than government-funded major
scientific agencies do.
2. For every one woman, there are six male scientists in private sector R&D
establishments. However, the ratio is four for one in major scientific agencies.
3. The bulk of scientists (in private and publicly funded organisations included) were in
‘Engineering Technology’ followed by the Medical Sciences and Natural Sciences.

Q Source: Current events

95. Swachh Survekshan monitors the performance of Swachh Bharat Abhiyan. Its most recent report was released
by

A. Ministry of Jal Shakti


B. Ministry of Rural Development
C. Ministry of Housing and Urban Affairs
D. Ministry of Home Affairs

Correct Answer : C

Answer Justification :

www.insightsactivelearn.com 78
Total Marks : 200
TEST - 3 (Textbook)
( INSTA Prelims Test Series 2021 )

Justification: Swachh Survekshan 2020 report was recently released by the Union Housing
and Urban Affairs Ministry.

Swachh Survekshan 2020 covered 4,242 cities, 62 cantonment boards and 92 Ganga towns.
This survey was carried out in 28 days.

This year the Ministry has released rankings based on the categorisation of cities on
population, instead of releasing overall rankings.

Performance of various cities:

Indore was ranked the cleanest city in the overall category this year followed by Surat
and Navi Mumbai.
Chhattisgarh is the cleanest state in the category of states with more than 100 urban
local bodies (ULBs).
Ahmedabad is India’s the cleanest Mega city.
New Delhi is the cleanest capital city.
Chhattisgarh’s Ambikapur is the cleanest smallest city.
Bengaluru wins the Best Self Sustainability award in the Mega city category.
Jharkhand is the cleanest state in the category of state with less than 100 Urban Local
Bodies (ULBs).

Q Source:
https://www.insightsonindia.com/2020/08/21/insights-daily-current-affairs-pib-summary-21-aug
ust-2020/

96. Consider the following statements about the National Centre for Financial Education (NCFE).
1. It is a Not for Profit Company.
2. It is being promoted by the Reserve Bank of India (RBI) and other financial market regulatory
institutions.

Select the correct answer using the codes below.


A. 1 only
B. 2 only
C. Both 1 and 2
D. None

Correct Answer : C

Answer Justification :

Justification: National Centre for Financial Education (NCFE) is a Not for Profit Company
promoted by Reserve Bank of India (RBI), Securities and Exchange Board of India (SEBI),
Insurance Regulatory and Development Authority of India (IRDAI) and Pension Fund
Regulatory and Development Authority (PFRDA).

www.insightsactivelearn.com 79
Total Marks : 200
TEST - 3 (Textbook)
( INSTA Prelims Test Series 2021 )

National Strategy for Financial Education 2020-2025 (NSFE):

The NSFE has been put together by the National Centre for Financial Education (NCFE) in
consultation with the four financial sector regulators (Reserve Bank of India, Securities and
Exchange Board of India, Insurance Regulatory and Development Authority of India and
Pension Fund Regulatory and Development Authority) and other relevant stakeholders.

Highlights of NSFE: 2020-25- Key recommendations:

1. Adopt a ‘5 C’ – Content, Capacity, Community, Communication and Collaboration


– approach to achieve the financial well-being of all Indians.
2. It has suggested financial literacy content for school children (including curriculum and
co-scholastic), teachers, young adults, women, new entrants at workplace/
entrepreneurs (MSMEs), senior citizens, persons with disabilities, illiterate people, etc.
3. Capacity development of various intermediaries, who can be involved in providing
financial literacy.
4. Develop a ‘Code of Conduct’ for financial education providers.
5. Community-led approaches should be evolved for disseminating financial literacy in a
sustainable manner.
6. A specific period in the year needs to be identified to disseminate financial literacy
messages on a large/ focused scale.
7. Integrate financial education content in the school curriculum, various professional and
vocational courses (undertaken by the Ministry of Skill Development and
Entrepreneurship) through their Sector Skilling Missions and B.Ed./M.Ed. programmes.
8. Adopt a robust ‘Monitoring and Evaluation Framework’ to assess progress made under
the strategy.

Q Source:
https://www.insightsonindia.com/2020/08/21/insights-daily-current-affairs-pib-summary-21-aug
ust-2020/

97. Consider the following about Brown Dwarfs.


1. They are usually much smaller than an average planet.
2. They are outside the solar system.
3. They are easy to trace via radio waves even as they do not emit any visible light.

Select the correct answer using the codes below.


A. 2 only
B. 2 and 3 only
C. 1 and 3 only
D. None of the above

Correct Answer : A

www.insightsactivelearn.com 80
Total Marks : 200
TEST - 3 (Textbook)
( INSTA Prelims Test Series 2021 )

Answer Justification :

Justification: Recently, Citizen Scientists and NOIR Lab facilities discovered almost 100
nearby cool Brown Dwarfs.

Brown dwarf are an astronomical object that is intermediate between a planet and a
star.
They usually have a mass less than 0.075 that of the Sun, or roughly 75 times that of
Jupiter.
They are outside the solar system, they give off little light and energy and they are
challenging to locate.
Brown dwarfs are also called failed stars, because their masses are heavier than planets
but lighter than stars.
Due to their small masses, they are unable to sustain fusion of their hydrogen to produce
energy.
It is believed that some of the more massive brown dwarfs fuse deuterium or lithium and
glow faintly.

Q Source: Current events

98. ‘Go’ and ‘No Go’ areas, with respect to forest resources, are mentioned in the

A. Wildlife Conservation Act, 1974


B. Forest Conservation Act, 1980
C. The Environment (Protection) Act, 1986
D. None of the above

Correct Answer : D

Answer Justification :

Justification: In 2009, the environment and coal ministries had jointly placed the country’s
forested areas under two categories – Go and No-Go – and imposed a ban on mining in the ‘No-
Go’ zones on environmental grounds.

‘No Go’ areas are those having either more than 10 per cent weighted forest cover (WFC) or
more than 30 per cent gross forest cover (GFC).

The exercise is aimed at prioritising forest areas under the Forest Conservation Act, 1980.

www.insightsactivelearn.com 81
Total Marks : 200
TEST - 3 (Textbook)
( INSTA Prelims Test Series 2021 )

Besides, Diversion of forest land for coal mining in these areas, which are rich in flora and
fauna, will have “avoidable serious adverse impact on forests and wildlife”.

If mining were to continue, even with afforestation and reclamation, it would not be possible to
restore the regions biodiversity.

The concept has no legal standing – They are mandated neither under Forest
Conservation Rules, 2003 nor under any circular issued by the ministry of
environment and forests.

Q Source:
https://www.insightsonindia.com/2020/08/19/insights-daily-current-affairs-pib-summary-19-aug
ust-2020/

99. Which one of these terms used in parliamentary proceedings is unrelated to the other three?

A. No-confidence motion
B. Floor test
C. Collective responsibility of the Council of Ministers to Lok Sabha
D. Zero Hour

Correct Answer : D

Answer Justification :

Justification: Option D is a question hour which is a part of the first session of the day. It is
reserved for asking questions of general concern to the government or a minister, and does
not necessarily concern collective responsibility.

Options A, B and C: A confidence motion, or a vote of confidence, or a trust vote, is sought by


the government in power on the floor of the House.

It enables the elected representatives to determine if the Council of Ministers commanded the
confidence of the House.

Floor test is a term used for the test of the majority.

If there are doubts against the chief minister, the governor can ask him to prove his
majority in the House.
In case of a coalition government, the chief minister may be asked to move a vote of
confidence and win a majority.

A no-confidence motion, or vote of no-confidence, or a no-trust vote, can be sought


by any House member to express that they no longer have confidence in the government.

This is because according to Article 75 (3) and Article 164 of the Constitution, the

www.insightsactivelearn.com 82
Total Marks : 200
TEST - 3 (Textbook)
( INSTA Prelims Test Series 2021 )

Council of Ministers are collectively responsible to the House of the People.

Q Source:
https://www.insightsonindia.com/2020/08/15/insights-daily-current-affairs-pib-summary-15-aug
ust-2020/

100. Consider the following statements.


The Constitution of India provides that
1. all proceedings in the Supreme Court and in every High court shall be in English Language
until Parliament by law otherwise provides.
2. only the President can authorize the use of Hindi language, or any other language used for any
official purpose of the State, in the proceedings of the High Court

Select the correct answer using the codes below.


A. 1 only
B. 2 only
C. Both 1 and 2
D. None

Correct Answer : A

Answer Justification :

Justification: Article 348 (1) of the Constitution of India provides that all proceedings in the
Supreme Court and in every High court shall be in English Language until Parliament by law
otherwise provides.

Under Article 348 (2), the Governor of the State may, with the previous consent of the
President, authorize the use of the Hindi language or any other language used for any official
purpose of the State, in the proceedings of the High Court having its principal seat in that
State provided that decrees, judgments or orders passed by such High Courts shall be in
English.

Other legal provisions:

Section 7 of the Official Languages Act, 1963, provides that the use of Hindi or official
language of a State in addition to the English language may be authorized, with the consent of
the President of India, by the Governor of the State for purpose of judgments etc. made by the
High Court for that State.

Q Source:
https://www.insightsonindia.com/2020/08/15/insights-daily-current-affairs-pib-summary-15-aug

www.insightsactivelearn.com 83

You might also like